Robbins and Cotran Review of Pathology - Pathology I

Réussis tes devoirs et examens dès maintenant avec Quizwiz!

At the site of a surgical incision, endothelial cells elaborate vascular endothelial growth factor. There is sprouting with migration of endothelial cells into the wound to establish new capillaries. Which of the following intracellular proteins is most important in facilitating movement of endothelial cells? A Actin B Cytokeratin C Desmin D Lamin E Myosin

A Actin Actin is a microfilament involved with cell movement. The other possibilities listed in B to D are intermediate fila- ments, which are larger than actin but smaller than myosin (a thick filament interdigitating with actin, required for mus- cle movement). Cytokeratins form cytoskeletal elements of epithelial cells. Desmin forms the scaffold in muscle cells on which actin and myosin contract. Lamin is associated with the nuclear membrane. (CH1)

A 26-year-old man with a family history of colon carcinoma undergoes a surveillance colonoscopy. It reveals hundreds of polyps in the colon, and two focal 0.5-cm ulcerated areas. A biopsy specimen from an ulcer reveals irregularly shaped glands that have penetrated into the muscular layer. Which of the following molecular events is believed to occur very early in the evolution of his colonic disease process? A Activation of the WNT signaling pathway B Inability to hydrolyze GTP-bound RAS C Loss of heterozygosity affecting the p53 gene D Mutations in mismatch repair genes. E Translocation of BCL2 from mitochondria to cytoplasm

A Activation of the WNT signaling pathway The patient has a classic history of familial adeno- matous polyposis with numerous adenomatous polyps and malignant transformation. The earliest event in the APC → adenocarcinoma sequence is loss of APC gene function. This prevents the destruction of β-catenin in the cytoplasm, which translocates to the nucleus and coactivates transcription of several genes. The APC → β-catenin sequence is a component of the WNT signaling pathway. RAS activation occurs after the malignant transformation sequence is initiated by the APC (gatekeeper) gene. Loss of cell cycle G1 arrest occurs with p53 loss late in the sequence. Mutations in mismatch repair genes give rise to hereditary nonpolyposis colon cancer syndrome from loss of ability to repair DNA damage. The BCL2 gene is not involved in the transition from adenoma to carcinoma. (CH7)

A 40-year-old man has a positive stool guaiac test during a routine physical examination. A colonoscopy is performed and a 0.9-cm, circumscribed, pedunculated mass on a short stalk is found in the upper rectum. Which of the following terms best describes this lesion? A Adenoma B Carcinoma C Choristoma D Hamartoma E Hyperplasia F Sarcoma

A Adenoma A discrete small mass such as that described is proba- bly benign. Adenomas arise from epithelial surfaces. Though adenocarcinoma may arise from a colonic adenoma, such ma- lignant lesions tend to be larger and more irregular. A choris- toma is a benign mass composed of tissues not found at the site of origin. A hamartoma is a rare benign mass composed of tissues usually found at the site of origin. A hyperplastic colonic lesion tends to be smaller and flatter. A sarcoma is a malignant neoplasm arising in mesenchymal tissues, not in epithelium. (CH7)

A 53-year-old man diagnosed with oral cancer and treated with radiation and chemotherapy 1 year ago now has a positron emission tomography (PET) scan of his neck that shows a single focus of increased uptake. This focus is resected and microscopic examination shows that it is a metastasis. Molecular analysis of this cancer shows p53, PTEN, and c-MYC gene mutations. Which of the following metabolic pathways is most likely up-regulated to promote his cancer cell survival and proliferation? A Aerobic glycolysis B Gluconeogenesis C Hexose monophosphate shunt D Oxidative phosphorylation E Purine degradation

A Aerobic glycolysis The PET scan is based upon selective uptake of a glu- cose derivative into tumor cells. The Warburg effect occurs when cancer cells shift their metabolism to aerobic glycolysis for selective growth advantage under harsh circumstances. Glycolysis also yields pyruvate for anabolic demands of increased tumor doublings. The p53 and c-MYC genes favor this metabolic change, whereas PTEN inhibits tumor cell autophagy, giving cancer cells an edge in growth. Cancer cells are less differentiated than normal cells and thus have decreased ability to do many complex biochemical processes, so they favor a simple one—glycolysis. Gluconeogenesis is a function of hepatocytes in response to decreased caloric intake. The HMP shunt and Krebs cycle are more useful to normal cells maintaining themselves at the status quo. Neoplasms generate large amounts of purines from cell divisions and cell turnover that must be eliminated as uric acid, but neoplastic cells do not perform this task. (CH7)

A 61-year-old man with a history of chronic viral hepatitis has noted a 6-kg weight loss over the past 5 months. Physical examination shows no masses or palpable lymphadenopathy. An abdominal CT scan shows a nodular liver with a 10-cm mass in the right lobe. A stool guaiac test result is negative. An elevation in which of the following laboratory tests is most likely to be present in this man? A Alpha-fetoprotein B CA-19-9 C Calcitonin D Carcinoembryonic antigen E Immunoglobulin M

A Alpha-fetoprotein Some chronic hepatitis B and C viral infections progress to hepatocellular and/or cholangiolar carcinoma. α-Fetoprotein (AFP) is an oncofetal protein that is a tumor marker for hepatocellular carcinomas and some testicular carcinomas. AFP is normally present in fetal life but not in adults. A serum immunoglobulin level with protein electrophoresis aids in the diagnosis of myeloma. Gastrointestinal tract adenocarcinomas, including those arising in the stomach, colon, and pancreas, as well as some lung carcinomas, may be accompanied by elevations in the serum carcinoembryonic antigen level. CA-19-9 is a tumor marker for colonic and pancreatic carcinomas. Some thyroid carcinomas produce calcitonin. Unfortunately, the sensitivity and specificity of tumor marker tests for detection of cancers, when they are small, is not high. (CH7)

A 54-year-old woman notes a lump in her right breast. Physical examination shows a 2-cm mass fixed to the under- lying tissues beneath the areola and three firm, nontender, lymph nodes palpable in the right axilla. There is no family history of cancer. An excisional breast biopsy is performed, and microscopic examination shows the findings in the figure. Over the next 6 months, additional lymph nodes become enlarged, and CT scans show nodules in the lung, liver, and brain. Which of the following molecular abnormalities is most likely to be found in her carcinoma cells? A Amplification of the ERBB2 (HER2) gene B Deletion of one RB gene copy C Fusion of BCR and C-ABL genes D Inactivation of one BRCA1 gene copy E Mutation of one p53 gene copy

A Amplification of the ERBB2 (HER2) gene Infiltrating ductal and intraductal carcinoma are present in the figure. Increased expression of ERBB2 (HER2) can be detected immunohistochemically and by fluorescence in situ hybridization (FISH) in the biopsy specimen. One third of breast cancers may show this change. Such amplification is associated with a poorer prognosis. Detection of a specific gene product in the tissue has value for determination of treatment and prognosis. BRCA1 and p53 mutations, if inherited in the germ line, can predispose the patient to breast cancer and other tumors. With BRCA1, there is family history of breast cancer, often at a young age. The tumor suppressor gene p53 mutations predispose to many types of cancers. An inherited deletion of RB gene predisposes to retinoblastoma. The BCR-ABL fusion product, seen in chronic myeloid leukemia, often results from t(9;22). (CH7)

A 57-year-old woman has experienced an increasing feeling of fullness in her neck along with a 3-kg (7-lb) weight loss over the past 3 months. On physical examination, there is a firm, fixed mass in a 3 × 5 cm area in the right side of the neck. A CT scan shows a solid infiltrating mass in the region of the right lobe of the thyroid gland. A biopsy of the mass is performed and the microscopic appearance is shown in the figure. All areas of the tumor have similar morphology. Which of the following terms best describes this neoplasm? A Anaplastic B Apoptotic C Dysplastic D Metaplastic E Well-differentiated

A Anaplastic The cells shown in the figure show marked pleomor- phism and hyperchromatism (anaplasia), and it is difficult to discern the cell of origin because no differentiation is noted. A bizarre tripolar mitotic figure is present. This degree of anaplasia is consistent with an aggressive, high-grade malignancy called anaplastic carcinoma. Apoptosis is single cell necrosis, but the cells shown appear viable and not frag- mented. Dysplasia refers to changes within an epithelium that presage a neoplasm. Metaplasia with one epithelial cell type substituted for another may presage dysplasia and malignancy. Well-differentiated neoplasms tend to be less aggressive and slower growing and resemble the cell of origin. (CH7)

An experiment involves factors controlling wound healing. Skin ulcerations are observed, and the factors involved in the healing process are analyzed. Which of the following factors is most likely to be effective in promoting angiogenesis? A Basic fibroblast growth factor B Endostatin C Epidermal growth factor D Interleukin-1 E Platelet-derived growth factor

A Basic fibroblast growth factor Basic fibroblast growth factor is a potent inducer of angiogenesis. It can participate in all steps of angiogenesis. Endostatin is an inhibitor of angiogenesis. Epidermal growth factor and interleukin-1 have no significant angiogenic activity. Platelet-derived growth factor plays a role in vascular remodeling. (CH1)

A 69-year-old man has noted a chronic cough for the past 3 months. On physical examination, there is mild stridor on inspiration over the right lung. A chest radiograph shows a 5-cm right hilar lung mass, and a fine-needle aspiration biopsy specimen of the mass shows cells consistent with squamous cell carcinoma. If staging of this neoplasm is denoted as T2N1M1, which of the following findings is most likely in this man? A Brain metastases B Elevated corticotropin level C Infiltration of the chest wall D Obstruction of a mainstem bronchus E Poorly differentiated tumor cells

A Brain metastases The M1 designation indicates that distant metasta- ses are present. N1 means local lymph nodes are positive for carcinoma. Elevated corticotropin levels indicate secretion of an ectopic hormone that may produce a paraneoplastic syndrome, but this is not part of staging. A T2 designation indicates that the overall size of the tumor is not large; it is still within the lung parenchyma and not impinging upon margins of the lung. The TNM system is used for staging, not microscopic grading of cellular differentiation. (CH7)

A clinical study involves patients diagnosed with carcinoma whose tumor stage is T4N1M1. The patients' survival rate 5 years from the time of diagnosis is less than 50%, regardless of therapy. Which of the following clinical findings is most likely to be characteristic of this group of patients? A Cachexia B Cardiac murmur C Icterus D Loss of sensation E Splenomegaly F Tympany

A Cachexia Cachexia is a common finding in advanced cancers, and weight loss without dieting in an adult is a "red flag" for malignancy. The exact cause for this is unknown, but increases in circulating factors such as tumor necrosis factor (TNF) may play a role. Cardiac murmurs may occur in the development of nonbacterial thrombotic endocarditis, a feature of a hypercoagulable state that may occur with advanced malignancies. Icterus is most likely to occur when there is obstruction of the biliary tract by a mass (e.g., as in pancreatic cancer), but metastases are unlikely to cause such an obstruction. Neurologic abnormalities may occur in local tumor growth impinging on nerves, but dull constant pain is the most likely abnormality in malignant neoplasms that invade nerves. Metastases to the spleen are uncommon. Tympany is uncommon in cancer because obstruction by a mass tends to be incomplete and to develop over a long time. (Hint: an empty beer keg is tympanitic when percussed.) (CH7)

During the past 6 months, a 25-year-old woman has noticed a malar skin rash that is made worse by sun exposure. She also has had arthralgias and myalgias. On physical examination, she is afebrile and has a pulse of 100/min, respirations of 20/min, and blood pressure of 100/60 mm Hg. There is erythema of skin over the bridge of the nose. No organomegaly is noted. Laboratory findings include positive serologic test results for ANA and double-stranded DNA, hemoglobin of 8.1 g/dL, hematocrit of 24.4%, platelet count of 87,000/mm3, and WBC count of 3950/mm3. The peripheral blood smear shows nucleated RBCs. A dipstick urinalysis is positive for blood, but there are no WBCs, RBCs, or casts seen on microscopic examination of the urine. Which of the following laboratory findings is most likely to be present? A Decreased haptoglobin B Decreased iron C Decreased reticulocytosis D Elevated D dimer E Elevated hemoglobin F F Elevated protoporphyrin

A Decreased haptoglobin Haptoglobin is a serum protein that binds to free hemoglobin. Ordinarily, circulating hemoglobin is contained within RBCs, but hemolysis can release free hemoglobin. The haptoglobin is used up as the amount of free hemoglobin increases. Systemic lupus erythematosus (SLE) is an autoimmune disease that can result in hemolysis by means of autoantibodies directed at RBCs, and the Coombs test result is often positive. SLE is best known to afflict young women, but it has a broad age range. Decreased iron can cause a hypochromic, microcytic anemia, but with hemolysis, the RBCs are recycled. Hemolysis is often accompanied by reticulocytosis if the marrow is intact and the iron is not lost. An elevated D-dimer level suggests a microangiopathic hemolytic anemia. Autoimmune diseases do not affect globin chain synthesis. Protoporphyrin can be increased with some forms of porphyria. (CH14)

A 40-year-old man notices an increasing number of lumps in his groin and armpit over the past 5 months. On physical examination, he has generalized nontender lymph node enlargement and hepatosplenomegaly. An inguinal lymph node biopsy specimen shows a malignant tumor of small, well-differentiated lymphoid cells. Immunostaining of the tumor cells with antibody to BCL2 is positive in the lymphocytic cell nuclei. Which of the following mechanisms has most likely produced this lymphoid neoplasm? A Diminished apoptosis B Gene amplifications C Increased tyrosine kinase activity D Loss of cell cycle inhibition E Reduced DNA repair

A Diminished apoptosis The BCL2 gene controls production of a protein that inhibits apoptosis, and overexpression of this gene allows accumulation of abnormal cells in lymphoid tissues. Gene amplifications typically affect the ERBB2 (HER2) and MYC oncogenes. Increased tyrosine kinase activity results from mutations affecting the ABL oncogene. Loss of cell cycle in- hibition results from loss of tumor suppressor genes such as p53. Reduced DNA repair occurs in the inherited disorder xeroderma pigmentosum. (CH7)

A 33-year-old woman with multiple sexual partners has had vaginal bleeding and discharge for the past 5 days. On physical examination, she is afebrile. Pelvic examination shows an ulcerated lesion arising from the squamocolumnar junction of the uterine cervix. A cervical biopsy is performed and microscopic examination reveals an invasive tumor containing areas of squamous epithelium, with pearls of keratin. In situ hybridization shows the presence of human papilloma-virus type 16 (HPV-16) DNA within the tumor cells. Which of the following molecular abnormalities in this tumor is most likely related to infection with HPV-16? A Functional inactivation of the RB protein B Increased expression of epidermal growth factor receptor C Epigenetic silencing of the RB gene D Inability to repair DNA damage E Trapping of the RAS protein in a GTP-bound state

A Functional inactivation of the RB protein The oncogenic potential of human papillomavirus (HPV), a sexually transmissible agent, is related to products of two early viral genes—E6 and E7. E7 protein binds to RB protein to cause displacement of normally sequestered tran- scription factors, which nullifies tumor suppressor activity of the RB protein. E6 protein binds to and inactivates the p53 gene product. Increased epidermal growth factor receptor expression is a feature seen in many pulmonary squamous cell carcinomas, and the related ERBB2 (HER2) receptor is seen in some breast carcinomas. Epigenetic modifications include DNA methylation and histone modifications which, depending on their nature, may enhance or inhibit gene expression. Inability to repair DNA damage plays a role in some colon and skin cancers. Trapping of GTP-bound RAS protein can occur in many tumors but is not related to HPV infection. (CH7)

An infant is born at 34 weeks' gestation to a 28-year-old woman, G3, P2. At birth, the infant is observed to be markedly hydropic and icteric. A cord blood sample is taken, and direct Coombs test result is positive for the infant's RBCs. Which of the following is the most likely mechanism for the findings in this infant? A Hemolysis of antibody-coated cells B Hematopoietic stem cell defect C Impaired globin synthesis D Mechanical fragmentation of RBCs E Oxidative injury to hemoglobin F Reduced deformability of RBC membranes

A Hemolysis of antibody-coated cells The infant most likely has erythroblastosis fetalis because maternal antibodies are coating fetal RBCs. A fetal- maternal hemorrhage in utero or at the time of delivery in a previous pregnancy (or with previous transfusion of incompatible blood) can sensitize the mother, resulting in production of irregular IgG antibodies. In subsequent pregnancies, these antibodies (in contrast to the naturally occurring IgM antibodies) can cross the placenta to attach to fetal cells, leading to hemolysis. In the past, most cases were caused by Rh incompatibility (e.g., Rh-negative mother, Rh-positive in- fant), but the use of RhoGAM administered at birth to Rh- negative mothers has eliminated almost all such cases when recognized. Other, less common blood group antigens can be involved in this process, however. The other conditions listed are not antibody mediated. A stem cell defect results in aplastic anemia and immunodeficiency. Impaired globin synthesis occurs in thalassemias. Mechanical fragmentation of RBCs is typical of microangiopathic hemolytic anemias, such as disseminated intravascular coagulation, which is more typical of pregnant women with obstetric complica- tions. Oxidative injury to hemoglobin is typical of glucose- 6-phosphate dehydrogenase (G6PD) deficiency. Reduced RBC membrane deformability is seen in patients with abnormalities in cytoskeletal proteins, such as spectrin; the latter causes hereditary spherocytosis. (CH14)

A 3-year-old boy from Sicily has a poor appetite and is underweight for his age and height. Physical examination shows hepatosplenomegaly. The hemoglobin concentration is 6 g/dL, and the peripheral blood smear shows severely hypochromic and microcytic RBCs. The total serum iron level is normal, and the reticulocyte count is 10%. A radiograph of the skull shows maxillofacial deformities and expanded marrow spaces. Which of the following is the most likely cause of this child's illness? A Imbalance in α-globin and β-globin chain production B Increased fragility of erythrocyte membranes C Reduced synthesis of hemoglobin F D Relative deficiency of vitamin B12 E Sequestration of iron in reticuloendothelial cells

A Imbalance in α-globin and β-globin chain production This patient of Mediterranean descent has β-thalassemia major. In this condition, there is a severe reduction in the synthesis of β-globin chains without impairment of α-globin synthesis. The free, unpaired α-globin chains form aggregates that precipitate within normoblasts and cause them to undergo apoptosis. The death of RBC precursors in the bone marrow is called "ineffective erythropoiesis." Not only does this cause anemia, but it also increases the absorption of dietary iron, giving rise to iron overload, which results in hemochromatosis with infiltrative cardiomyopathy, hepatic cirrhosis, and "bronze diabetes" from pancreatic islet dysfunction. The severe anemia triggers erythropoietin synthesis, which expands the erythropoietic marrow. The marrow expansion encroaches on the bones, causing maxillofacial deformities. Extramedullary hematopoiesis causes hepatosplenomegaly. In comparison, the hemolytic anemia is mild in β-thalassemia minor, and there is very little ineffective erythropoiesis. Hemochromatosis is particularly detrimental to the liver and heart. Patients with chronic anemia may require RBC transfusions, which adds even more iron to body stores. The other listed options do not lead to a marked expansion of hematopoiesis. (CH14)

A study of peripheral blood smears shows that neutrophil nuclei of women have a Barr body, whereas those of men do not. The Barr body is an inactivated X chromosome. Which of the following forms of RNA is most likely to play a role in Barr body formation? A lncRNA B mRNA C miRNA D siRNA E tRNA

A lncRNA There are forms of noncoding RNA that play a role in gene expression. Long noncoding RNA (lncRNA) segments greater than 200 nucleotides in length can bind to chromatin to restrict access of RNA polymerase to coding segments. The X chromosome transcribes XIST, a lncRNA that binds to and represses X chromosome expression. However, not all genes on the "inactive" X chromosome are switched off. The RNA transcribed from nuclear DNA that directs protein syn- thesis through translation is mRNA. MicroRNAs (miRNAs) are noncoding RNA sequences that inhibit the translation of mRNAs. Gene-silencing RNAs (small interfering RNAs [siRNAs]) have the same function as miRNAs, but they are produced synthetically for experimental purposes. Transfer RNA (tRNA) participates in the translation of mRNA to pro- teins by linking to specific amino acids. (CH1)

A 49-year-old man has a lump near his right shoulder that has been increasing in size for the past 8 months. On physical examination, a 4-cm, firm, nontender mass is palpable in the right supraclavicular region. The mass is excised, and microscopically it shows a lymphoid neoplasm. Karyotypic analysis of the cells shows a chromosomal translocation, t(14;18), bringing the immunoglobulin heavy chain gene together with another gene. Which of the following genes is most likely activated by this translocation? A APC (tumor suppressor gene) B BCL2 (anti-apoptosis gene) C BRCA1 (DNA repair gene) D c-MYC (transcription factor gene) E IL2 (growth factor gene) F K-RAS (GTP-binding protein gene)

B BCL2 (anti-apoptosis gene) This is an example of chromosomal translocation that brings BCL2, an anti-apoptosis gene, close to another gene (immunoglobulin heavy chain gene). The BCL2 gene becomes subject to continuous stimulation by the adjacent enhancer element of the immunoglobulin gene, leading to overexpression. The APC gene is mutated in sporadic colon cancers and cancers associated with familial polyposis coli. The BRCA1 gene mutation is seen in some breast cancers. The c-MYC gene is found on chromosome 8, and the t(8;14) translocation seen in many Burkitt lymphomas leads to MYC overexpression. The IL2 mutation may be present in some T cell neoplasms. K-RAS mutations are present in many cancers, but not typically lymphoid malignancies. (CH7)

A 26-year-old woman has a lump in her left breast. On physical examination, she has an irregular, firm, 2-cm mass in the upper inner quadrant of the breast. No axillary adenopathy is noted. A fine-needle aspirate of the mass shows anaplastic ductal cells. The patient's 30-year-old sister was recently diagnosed with ovarian cancer, and 3 years ago her maternal aunt was diagnosed with ductal carcinoma of the breast and had a mastectomy. Mutation involving which of the following genes is most likely present in this family? A BCL2 (anti-apoptosis gene) B BRCA1 (DNA repair gene) D ERBB2 (growth factor receptor gene) E HST1 (fibroblast growth factor gene) F IL2 (growth factor gene) G K-RAS (GTP-binding protein gene)

B BRCA1 (DNA repair gene) Approximately 5% to 10% of breast cancers are familial, and 80% of these cases result from mutations in the BRCA1 and BRCA2 genes. Onset of these familial cancers occurs earlier in life than the sporadic cancers. The protein products of these genes are involved in DNA repair. BCL2 is overexpressed in some lymphoid neoplasms. The epithelial growth factor receptor ERBB2 (HER2) overexpression is present in some sporadic breast cancers; other EGF alterations can be seen in lung, bladder, gastrointestinal, ovarian, and brain neoplasms. The HST1 mutation is seen in some gastric cancers. IL2 overexpression is associated with some T cell neoplasms. K-RAS overexpression is seen in many cancers, including some breast cancers, but the early age of onset and family history in this case strongly suggest BRCA mutations.

In an experiment, a nuclear chromosomal gene is found to be actively transcribing messenger RNA (mRNA) that is transported into the cell cytoplasm. However, there is no observed protein product from translation of this mRNA. How is the silencing of this active gene's mRNA most likely to occur? A Absence of tRNA B Binding to miRNA C Methylation of DNA D Mutation of mRNA E Upregulation of mtDNA

B Binding to miRNA MicroRNAs (miRNA) are encoded by about 5% of the human genome. miRNAs do not encode for proteins, but bind to and inactivate or cleave to mRNA, preventing translation of proteins by mRNA, effectively silencing gene expression without affecting the gene directly. There is abundant tRNA present in the cytoplasm that is not a rate- limiting step to translation. DNA methylation, particularly at CG dinucleotides, is a way of suppressing gene expres- sion directly, as is seen with genomic imprinting. Mutations that occur in genes in DNA may result in reduced mRNA production or abnormal protein production, but mRNA it- self is not mutated. Mitochondrial DNA (mtDNA) encodes for proteins mainly involved in oxidative phosphorylation metabolic pathways. (CH1)

In a study of patients with non-Hodgkin B cell lymphoma, a nuclear gene is found to be actively transcribed to mRNA and is transported into the cell cytoplasm. A protein is translated from this mRNA, with up-regulation of BCL2. In a control group without lymphoma, translation of the mRNA does not occur. How is the silencing of this active gene's mRNA most likely to occur? A Absence of tRNA B Binding to miRNA C Methylation of DNA D Mutation of mRNA E Up-regulation of mtDNA

B Binding to miRNA MicroRNAs (miRNAs) are encoded by about 5% of the human genome. miRNAs do not encode for proteins, but bind and inactivate or cleave mRNA, preventing translation of proteins by mRNA. This effectively silences gene expression without affecting the gene directly. There is abundant transfer RNA (tRNA) present in the cytoplasm that is not a rate-limiting step to translation. DNA methylation, particularly at CG dinucleotides, is a way of suppressing gene expression directly, as is seen with genomic imprinting. Mutations that occur in genes in DNA may result in reduced mRNA production or abnormal protein production, but mRNA itself is not mutated. Mitochondrial DNA (mtDNA) encodes for proteins that are mainly involved in oxidative phosphorylation metabolic pathways. (CH7)

An epidemiologic study of cancer deaths recorded in the last half of the 20th century is conducted. The number of deaths for one particular cancer had increased markedly in developed nations. More than 30% of cancer deaths in men, and more than 24% of cancer deaths in women, were caused by this neoplasm in 1998. In some nations, prevention strategies reduced deaths from this cancer. Which of the following neoplasms was most likely identified by this study? A Cerebral glioma B Bronchogenic carcinoma C Hepatocellular carcinoma D Colonic adenocarcinoma E Pancreatic adenocarcinoma F Skin melanoma

B Bronchogenic carcinoma Incidence of lung cancers increased dramatically in the 20th century because of the popularity of cigarette smoking. As the number of individuals in a population who smoke increases, so do the number of lung cancers. Some cancers of the urinary tract, oral cavity, esophagus, and pan- creas also are causally related to smoking. Breast, prostate, and colon cancers remain common in developed nations, but the number of cases has not increased sharply. Pap smear screening and human papillomavirus (HPV) vaccination markedly decreases numbers of cervical cancers. There has been an increase in the incidence of melanomas worldwide, but there are still far fewer cases of melanomas than of lung cancers. Hepatic and intracranial neoplasms in adults are far less common than lung cancers. (CH7)

A study of colonic polyps is performed. Malignant cells localized to the polyp are compared to those from polyps showing invasion of the stalk. Molecular analysis shows up-regulation of certain molecules in the invasive malignant cells. Invasive lesions are more likely to exhibit lymphatic metastases. Which of the following markers is most likely to have increased expression in the invasive malignant epithelial cells? A BCL2 B CD44 C EGFR D RAS E Vimentin

B CD44 Malignant transformation includes many genetic changes, including those rendering the malignant cells capable of invasion and metastases. CD44 plays a role in cell adhesion and enables malignant cells to metastasize. Solid tumors can express CD44 to enhance their spread to lymph nodes and other metastatic sites. Though such properties may have been present with the initial clone of malignant cells, the growth of the cancer increases the number of cells and the risk for spread. BCL2 plays a role in apoptosis. Growth factor receptor expression may make malignant cells susceptible to environmental influences, such as hormones, that drive growth. RAS gene mutations are present in many cancers and lead to loss of growth control. Vimentin is best known as a protein expressed in mesenchymal neoplasms, such as sarcomas, but it is also up-regulated in EMT. (CH7)

A 62-year-old man has had several episodes of hematuria in the past week. He has a 48 pack-year history of smoking cigarettes. On physical examination, there are no abnormal findings. A urinalysis shows 4+ hematuria, and cytologic examination of the urine shows that atypical cells are present. A cystoscopy is performed and a 4-cm sessile mass with a nodular, ulcerated surface is seen in the dome of the bladder. Which of the following terms best describes this lesion? A Adenoma B Carcinoma C Fibroma D Papilloma E Sarcoma

B Carcinoma A large, irregular, ulcerated mass such as that de- scribed is most likely malignant, and the epithelium of the bladder gives rise to carcinomas. Urothelial carcinomas are associated with smoking. An adenoma is a benign epithelial neoplasm of glandular tissues. A fibroma is a benign mesen- chymal neoplasm. A papilloma is a benign, localized mass that has an exophytic growth pattern. A sarcoma is derived from cells of mesenchymal origin; sarcomas are much less common than carcinomas. (CH7)

A 22-year-old woman has experienced malaise and a sore throat for 2 weeks. Her fingers turn white on exposure to cold. On physical examination, she has a temperature of 37.8° C, and the pharynx is erythematous. Laboratory find- ings include a positive monospot (heterophile antibody) test result. Direct and indirect Coombs test results are positive at 4° C, although not at 37° C. Which of the following molecules bound on the surfaces of the RBCs most likely accounts for these findings? A α2-Macroglobulin B Complement C3b C Fibronectin D Histamine E IgE

B Complement C3b Cold agglutinin disease has antibody (usually IgM) coating RBCs. The IgM antibodies bind to the RBCs at low temperature at peripheral body sites and fix complement; however, complement is not lytic at this temperature. With an increase in temperature within core internal organs, the IgM is dissociated from the cell, leaving behind C3b. Most of the hemolysis occurs extravascularly in the cells of the mononuclear phagocyte system, such as Kupffer cells in the liver, or splenic macrophages, because the coating of complement C3b acts as an opsonin. IgG is typically involved in warm antibody hemolytic anemia, which is chronic and is not triggered by cold. Raynaud phenom- enon occurs in exposed, colder areas of the body, such as the fingers and toes. The patient probably has an elevated cold agglutinin titer. Histamine is released in type I hypersensitivity reactions. Fibronectin is an adhesive cell surface glycoprotein that aids in tissue healing. IgE is present in allergic conditions. (CH14)

A 49-year-old man experiences an episode of hemoptysis. On physical examination, he has puffiness of the face, pedal edema, and systolic hypertension. A chest radiograph shows an irregular perihilar 5-cm mass of the right lung. Laboratory studies show normal serum electrolytes. A transbronchial biopsy is performed, and the microscopic findings are shown in the figure. A bone scan shows no metastases. Immunohistochemical staining of the tumor cells is most likely to be positive for which of the following? A Antidiuretic hormone B Corticotropin C Erythropoietin D Insulin E Parathyroid hormone-related peptide

B Corticotropin The small cells have scant cytoplasm but marked hyperchromatism, consistent with small cell anaplastic carcinoma. This patient has Cushing syndrome resulting from ectopic corticotropin production by the tumor, a form of paraneoplastic syndrome common to small cell carcinomas of the lung. Such small cell carcinomas are of neuroendocrine derivation. A syndrome of inappropriate antidiuretic hormone (SIADH) secretion from small cell carcinomas is also common, but leads to hyponatremia as well as edema. Erythropoietin production with polycythemia is more likely to be associated with a renal cell carcinoma. Insulin and gastrin production are most often seen in islet cell tumors of the pancreas. Hypercalcemia from a parathyroid hormone-related peptide (PTHrP) is more typically associated with pulmonary squamous cell carcinomas. (CH7)

In an experiment, the role of low-density lipoprotein (LDL) receptors in uptake of lipids in the liver is studied. A mouse model is created in which the LDL receptor gene is not expressed in the liver. For creating such a knockout mouse, which of the following cells would be most useful? A Adult bone marrow mesenchymal progenitor cells B Embryonic stem cells in culture C Hematopoietic stem cells D Hepatic oval cells E Regenerating hepatocytes

B Embryonic stem cells in culture Embryonic stem (ES) cells are multipotent and can give rise to all cells, including hepatocytes. Gene targeting to produce knockout mice is done in cultures of ES cells, which are then injected into mouse blastocysts and implanted into the uterus of a surrogate mother. Mesenchymal stem cells also are multipotential, but they are not useful for gene tar- geting. Hematopoietic stem cells can give rise to all hema- topoietic cells, but not other types of cells. Hepatocytes and oval cells within the liver can give rise only to liver cells. (CH1)

A 34-year-old sexually active woman undergoes a rou- tine physical examination. There are no abnormal findings. A Pap smear is obtained as part of the pelvic examination. Cytologically, the cells obtained on the smear from the cervix show severe epithelial dysplasia (high-grade squamous intraepithelial lesion). Which of the following therapeutic options is most appropriate for this woman? A Antibiotic therapy B Excision C Ovarian removal D Screening of family members E Watchful waiting

B Excision Epithelial dysplasias, especially severe dysplasias, can be precursors of carcinomas. This is a key reason for Pap smear screening. The incidence of cervical carcinoma decreases when routine Pap smears are performed. Colposcopy with biopsy is indicated to determine the extent of the lesion for removal. Though related to human papillomavirus (HPV) infection, severe dysplasias are not amenable to anti- biotic therapy. Ovarian neoplasms are not related to cervical dysplasias or carcinomas. In general, cervical cancers are not related to hereditary syndromes, and cervical dysplasias are not hereditary. Screening of family members is appropriate for those who have risk factors, such as multiple sexual partners. Regression of a severe dysplasia is unlikely. (CH7)

An experiment is conducted involving cellular aspects of wound healing. Components of the extracellular matrix are analyzed to determine their sites of production and their binding patterns to other tissue components. Which of the following molecules synthesized by fibroblasts can best bind to cellular integrins and extracellular collagen and attach epidermal basal cells to basement membrane? A Dermatan sulfate B Fibronectin C Heparin D Hyaluronic acid E Procollagen

B Fibronectin Fibronectin is a key component of the extracellular matrix and has a structure that looks like a paper clip. Fibronectin can be synthesized by monocytes, fibroblasts, and endothelium. Dermatan sulfate, a glycosaminoglycan, forms a gel that provides resilience and lubrication. Heparin that is infused has an anticoagulant function. Hyaluronic acid binds water to form a gelatinous extracellular matrix. Procollagen produced by fibroblasts is formed into ropelike strands of collagen, which provide tensile strength. (CH1)

An epidemiologic study of cancer deaths recorded in the last half of the 20th century is conducted. The number of deaths for one particular type of cancer had been decreas-ing in developed nations, despite the absence of widespread screening and prevention programs. Which of the following neoplasms was most likely to be identified by this study? A Cerebral glioma B Gastric adenocarcinoma C Hepatic angiosarcoma D Leukemia E Pulmonary small cell carcinoma

B Gastric adenocarcinoma The decrease in the number of gastric cancers may be related to reduced numbers of dietary carcinogens or a decrease in the prevalence of Helicobacter pylori infection; however, the exact reason is obscure. Cerebral gliomas are not as common as carcinomas; an urban legend links them to cell phone use, but legitimate epidemiologic studies have not made this link. Angiosarcomas of the liver are quite rare; they are epidemio- logically linked to vinyl chloride exposure. Leukemias and lym- phomas are not as common as carcinomas. Pulmonary small cell carcinomas are related to smoking, and the numbers have decreased in many countries with campaigns to reduce smok- ing; the death rate is typically high because the prognosis for lung cancer is so poor. (CH7)

A 33-year-old man has experienced occasional head- aches for the past 3 months. He suddenly has a generalized seizure. CT scan of the head shows a periventricular 3-cm mass in the region of the right thalamus. A stereotactic biopsy of the mass yields large lymphoid cells positive for B cell markers. Which of the following underlying diseases is most likely to be found in this patient? A Diabetes mellitus B HIV infection C Hypertension B D Multiple sclerosis C E Tuberculosis

B HIV infection Primary or secondary immunodeficiency diseases carry an increased risk of neoplasia, particularly lymphomas. B cell lymphomas of the brain are 1000-fold more common in patients with AIDS from HIV infection than in the general population. Patients with diabetes mellitus can experience various vascular and infectious complications, although not neoplasia. Hypertension can lead to central nervous system hemorrhages (strokes). Multiple sclerosis is a demyelinating disease of CNS white matter and carries no significant risk of neoplasia. Tuberculosis as a chronic infection may lead to amyloidosis, not neoplasia. (CH7)

A 23-year-old woman has noted a nodule on the skin of her upper chest. She reports that the nodule has been present for many years and has not changed in size. It is excised, and the microscopic appearance is shown in the figure. Which of the following neoplasms is this lesion most likely to be? A Fibroadenoma B Hemangioma C Leiomyoma D Lipoma E Melanoma F Nevus

B Hemangioma The small, discrete nature of this mass and its slow growth with nearly unchanged size suggest a benign neo- plasm. The red color is consistent with vascularity. A heman- gioma is a common benign lesion of the skin. Fibroadenomas arise in the breast. Leiomyomas, which are white, arise from smooth muscle and are most common in the uterus. Lipomas are yellow fatty tumors that can occur beneath the epider- mis. Melanomas are malignant and tend to increase in size quickly; many are darkly pigmented. The benign counter- part to the melanoma is the nevus, which is quite common, but nevi are usually light to dark brown. (CH7)

An investigational study reviews cells harvested from patients 30 to 50 years of age who had right-sided colon cancer with multiple polyps present. These patients typically develop multiple malignant lesions of the colon during middle age. Molecular analysis of the cells from the lesions shows changes in hPMS1, hPMS2, and hMLH1 genes. Which of the following principles of carcinogenesis is best illustrated by this study? A Carcinogenesis is a multistep process B Inability to repair DNA is carcinogenic C Many oncogenes are activated by translocations D Tumor initiators are mutagenic E Tumor promoters induce proliferation

B Inability to repair DNA is carcinogenic Patientswithhereditarynonpolyposiscoloncarcinoma (HNPCC) inherit one defective copy of mismatch repair genes. Several human mismatch repair genes are involved in the development of HNPCC. Mismatch repair defects have microsatellite instability. Microsatellites are tandem repeats found through- out the genome. Normally, the length of these microsatellites remains constant. In HNPCC, these satellites are unstable and increase or decrease in length. Although HNPCC accounts for only 2% to 4% of all colon carcinomas, microsatellite instability can also be detected in about 15% of all sporadic colon carcino- mas. Mutations in mismatch repair genes can be detected by the presence of microsatellite instability. The other listed options are not characteristic of HNPCC. (CH7)

A 29-year-old rugby player takes part in a particularly contentious game between New Zealand and South Africa. He is the forward prop in the scrums, hitting hard and being hit hard by other players. He feels better after downing several pints of beer following the game, but notes darker urine. Urinalysis is positive for blood. Which of the following pathogenic mechanisms underlies change in the color of urine? A Complement lysis B Intravascular disruption C Osmotic fragility D Sinusoidal sickling E Splenic sequestration

B Intravascular disruption Mechanical trauma to RBCs is possible, but typically is not severe. It can follow strenuous exercises involving repeated blows to body parts. Complement-mediated lysis is a feature of immunohemolytic anemias. Increased osmotic fragility is noted in spherocytes. Sickle cell anemia is not likely to be found in the population groups in the countries noted, and persons with this disease are not likely to be playing rugby. Splenic sequestration is a feature of hemolytic anemias due to membrane defects and antibodies. (CH14)

An epidemiologic study analyzes health care benefits of cancer screening techniques applied to persons more than 50 years of age. Which of the following diagnostic screening techniques used in health care is most likely to have the great- est impact on reduction in cancer deaths in Europe and North America? A Chest radiograph B Mammography C Pap smear D Serum tumor markers E Stool guaiac F Urine cytology

B Mammography A screening program should reliably detect early cancers with higher incidence. Breast cancer affects up to 1 in 9 women in these regions. Mammography may aid in de- tection of small cancers that have a better prognosis. A chest radiograph is an insensitive technique for detecting early lung cancers. Because Pap smear screening can detect dys- plasias and in situ carcinomas that can be treated before pro- gression to invasive lesions, deaths from cervical carcinoma have steadily decreased since this screening method became widely available in the last half of the 20th century. The in- troduction of human papillomavirus (HPV) vaccination will diminish the numbers of cervical cancers even further. Serum tumor markers have not proved useful as general screening techniques, although they are useful in selected circumstanc- es. Use of stool guaiac has had a minimal effect on rates of death from colorectal carcinomas, but physicians are cau- tioned not to indicate "rectal deferred" on the physical ex- amination report, and hence contribute to the problem. Urine cytology is better than urinalysis for detection of urothelial malignancies, but it does not have a high sensitivity. (CH7)

A 17-year-old girl has had a history of fatigue and weak- ness for her entire life. She has not undergone puberty. On physical examination, secondary sex characteristics are not well developed. She has hepatosplenomegaly. CBC shows hemoglobin of 9.1 g/dL, hematocrit of 26.7%, MCV of 66 μm3, platelet count of 89,000/mm3, and WBC count of 3670/mm3. The appearance of the peripheral blood smear is shown in the figure. Additional laboratory findings include serum glucose of 144 mg/dL, TSH of 6.2 mU/mL, and ferritin of 679 ng/mL. A mutation in a gene encoding for which of the following is most likely to be present in this girl? A Ankyrin B β-Globin C G6PD D HFE E NADPH oxidase

B β-Globin This patient has β-thalassemia, probably of at least intermediate severity. There is decreased β-globin chain formation, with increased hemoglobin A2 and F to compensate. There is ineffective erythropoiesis and increased erythropoietin to drive increased iron absorption, leading to iron overload. Chronic anemia requiring transfusion therapy exacerbates hemochromatosis. Iron deposited in endocrine tissues can lead to gonadal, pituitary, thyroid, islet cell, and adrenal failure. Secondary hypersplenism can result from the splenomegaly, with sequestration of platelets and leukocytes. The abnormal ankyrin gene leads to hereditary spherocytosis and a mild hemolytic anemia with splenomegaly, but not to iron overload. In glucose-6-phosphate dehydrogenase deficiency, sensitivity to oxidizing agents causes a hemolytic anemia, but this usually is not ongoing. The HFE gene is abnormal in hereditary hemochromatosis, leading to iron overload, but onset of the disease occurs in middle age. Mutations involving NADPH oxidase lead to immunodeficiency in chronic granulomatous disease. (CH14)

A 65-year-old man has experienced worsening fatigue for the past 5 months. On physical examination, he is afebrile and has a pulse of 91/min, respirations of 18/min, and blood pressure of 105/60 mm Hg. There is no organomegaly. A stool sample is positive for occult blood. Laboratory findings include hemoglobin of 5.9 g/dL, hematocrit of 18.3%, MCV of 99 μm3, platelet count of 250,000/mm3, and WBC count of 7800/mm3. The reticulocyte concentration is 3.9%. No fibrin split products are detected, and direct and indirect Coombs test results are negative. A bone marrow biopsy specimen shows marked erythroid hyperplasia. Which of the following conditions best explains these findings? A Aplastic anemia B Autoimmune hemolytic anemia C Chronic blood loss D Iron deficiency anemia E Metastatic carcinoma

C Chronic blood loss The marked reticulocytosis and marrow hyperplasia indicate that the marrow is responding to a decrease in RBCs. The reticulocytes are larger RBCs that slightly increase the MCV. An aplastic marrow is very hypocellular and unable to respond to anemia; it is associated with pancytopenia. The normal Coombs test results exclude an autoimmune hemolytic anemia. Iron deficiency impairs the ability of the marrow to mount a significant and sustained reticulocytosis. Iron deficiency anemia is typically microcytic and hypochromic, but could be partially masked here by reticulocytosis, which would not be as marked if iron were not available, but his diet is supplying needed iron. Infiltrative disorders, such as metastases in the marrow, would impair the ability to mount a reticulocytosis of this degree. (CH14)

A clinical study of patients who inherit mutations that reduce the level of ankyrin, the principal binding site for spectrin, in the RBC membrane cytoskeleton shows an increased prevalence of chronic anemia with splenomegaly. For many patients, it is observed that splenectomy reduces the severity of anemia. This beneficial effect of splenectomy is most likely related to which of the following processes? A Decrease in opsonization of RBCs and lysis in spleen B Decrease in production of reactive oxygen species by splenic macrophages C Decrease in splenic RBC sequestration and lysis D Increase in deformability of RBCs within splenic sinusoids E Increase in splenic storage of iron

C Decrease in splenic RBC sequestration and lysis In patients with hereditary spherocytosis, spheroidal cells are trapped and destroyed in the spleen because the abnormal RBCs have reduced deformability. Splenectomy is beneficial because the spherocytes are no longer detained by the spleen. Splenectomy has no effect on the synthesis of spectrin or RBC deformability; the RBCs in spherocytosis are not killed by opsonization. In warm antibody hemolytic ane- mias, opsonized RBCs are removed by the spleen. Reactive oxygen species do not play a role in anemias. Iron is not the rate-limiting step to RBC production when the iron can be recycled within the body. (CH14)

A 48-year-old woman notices a lump in her left breast. On physical examination she has a firm, nonmovable, 2-cm mass in the upper outer quadrant of the left breast. There are enlarged, firm, nontender lymph nodes in the left axilla. A fine-needle aspiration biopsy is performed, and the cells present are consistent with carcinoma. A lumpectomy with axillary lymph node dissection is performed, and carcinoma is present in two of eight axillary nodes. Reduced expression of which of the following molecules by the tumor cells is most likely responsible for the lymph node metastases? A Estrogen receptors B ERBB2 (HER-2) C E-cadherin D Progesterone receptors E Tyrosine kinases

C E-cadherin Several pathologic mechanisms play a role in the development of tumor metastases. The tumor cells first must become discohesive and detach from the primary site, degrade the basement membrane and interstitial connective tissue, and then attach elsewhere to become metastases. Reduced expression of adhesion molecules such as E-cadherins promotes metastases. Tumor cells can elaborate, not reduce, proteases such as metalloproteinases to promote discohesiveness. Expression of estrogen and progesterone receptors predicts breast cancer responsiveness to antihormone therapy, and there is a monoclonal antibody, trastuzumab, that targets HER-2, a form of epidermal growth factor receptor. Tyrosine kinase receptors within cells aid in signaling cell growth. (CH7)

A 13-year-old boy has the sudden onset of severe abdominal pain and cramping accompanied by chest pain, non-productive cough, and fever. On physical examination, his temperature is 39° C, pulse is 110/min, respirations are 22/min, and blood pressure is 80/50 mm Hg. He has diffuse abdominal tenderness, but no masses or organomegaly. Laboratory studies show a hematocrit of 18%. The peripheral blood smear is shown in the figure. A chest radiograph shows bilateral pulmonary infiltrates. Which of the following is the most likely mechanism for initiation of his pulmonary problems? A Chronic hypoxia of the pulmonary parenchyma B Defects in the alternative pathway of complement activation C Extensive RBC adhesion to endothelium D Formation of autoantibodies to alveolar basement membrane E Intravascular antibody-induced hemolysis

C Extensive RBC adhesion to endothelium The crescent-shaped RBCs (sickled RBCs) are char- acteristic of hemoglobin SS. This disease is most common in individuals of African and eastern Arabian descent. The sickled RBCs are susceptible to hemolysis (mainly vascular, in the spleen), but they also can cause microvascular occlusions anywhere in the body, most commonly bone, lungs, liver, and brain, leading to ischemia and severe pain. Vascular occlusions in the lungs are often accompanied by infection and lead to "acute chest syndrome." Abdominal pain and back pain are common and severe, requiring prompt and effective analgesia. The cell membranes of reversibly sickled cells are abnormally "sticky," and they adhere to capillary endothelium, especially in lungs. Vasoconstriction is caused by depletion of NO by free hemoglobin. Adhesion of RBCs to endothelium retards blood flow, creates hypoxia, and precipitates local sickling and vascular occlusion. Chronic tissue hypoxia does occur in sickle cell anemia, but it produces insidious impairment of function in organs such as heart, kidneys, and lungs. Defects in the alter- native pathway of complement activation predispose to infection with encapsulated bacteria, such as Haemophilus influenzae and Streptococcus pneumoniae. Autoantibodies to alveolar basement membrane can be part of Goodpasture syndrome, which also affects kidneys. The most severe intravascular hemolysis occurs with major transfusion reactions. (CH14)

A 12-year-old girl and a 14-year-old boy have developed skin nodules in predominantly sun-exposed areas of their skin over the past 5 years, but their six siblings have not. On physical examination, both children are of appropriate height and weight. The skin lesions are 1- to 3-cm maculopapular nodules that are erythematous to brown-colored and have areas of ulceration. Microscopic analysis of biopsy specimens of the skin lesions shows squamous cell carcinoma. The children have no history of recurrent infections, and their parents and other relatives are unaffected. Which of the following mechanisms is most likely to produce neoplasia in these children? A Inherited mutation of the p53 gene B Chromosomal translocation C Failure of nucleotide excision repair of DNA D Ingestion of food contaminated with Aspergillus flavus E Infection with human papillomavirus

C Failure of nucleotide excision repair of DNA These children have an autosomal recessive condition known as xeroderma pigmentosum (XP). Affected individuals have extreme photosensitivity, with a 2000-fold increase in the risk of skin cancers. The DNA damage is initiated by exposure to ultraviolet light; however, nucleotide excision repair cannot occur normally in XP because inheritance of one of several XP genes is abnormal. The inheritance pattern appears to be autosomal recessive, because new mutations are unlikely to occur. Inactivation of the p53 tumor suppressor gene is found in many sporadic human cancers and in some familial cancers, but these cancers are not limited to the skin. Chromosomal translocations are often involved in the development of hematologic malignancies, although they are not often seen in skin cancers. Aspergillus flavus, found on moldy peanuts and other foods, produces the potent hepatic carcinogen aflatoxin B1. Human papillomavirus (HPV) is a sexually transmitted disease that is associated with the de- velopment of genital squamous cell carcinomas. (CH7)

A 23-year-old African-American man passes dark red- dish brown urine 3 days after taking an anti-inflammatory medication that includes phenacetin. He is surprised, because he has been healthy all his life and has had no major illnesses. On physical examination, he is afebrile, and there are no remarkable findings. CBC shows a mild normocytic anemia, but the peripheral blood smear shows precipitates of denatured globin (Heinz bodies) with supravital staining and scattered "bite cells" in the population of RBCs. Which of the following is the most likely diagnosis? A α-Thalassemia minor B β-Thalassemia minor C Glucose-6-phosphate dehydrogenase deficiency D Sickle cell trait E Abnormal ankyrin in RBC cytoskeletal membrane F Warm antibody autoimmune hemolytic anemia

C Glucose-6-phosphate dehydrogenase deficiency Glucose-6-phosphate dehydrogenase (G6PD) deficiency is an X-linked disorder that affects about 10% of African- American males. The lack of this enzyme subjects hemoglobin to damage by oxidants, including drugs such as primaquine, sulfonamides, nitrofurantoin, phenacetin, and aspirin (in large doses). Infection can also cause oxidative damage to he- moglobin. Heinz bodies are denatured hemoglobin, and they damage the RBC membrane, giving rise to intravascular hemolysis. The "bite cells" result from the attempts of overeager splenic macrophages to pluck out the Heinz bodies, adding an element of extravascular hemolysis. Heterozygotes with α-thalassemia (1 or 2 abnormal genes out of 4 total α-globin genes) have no major problems, but in cases of α-thalassemia major, perinatal death is the rule. Likewise, β-thalassemia minor and sickle cell trait are conditions usually with no major problems and no relation to drug usage. RBC membrane abnormalities, such as hereditary spherocytosis (caused by abnormal spectrin), typically produce a mild anemia without significant hemolysis, and there is no drug sensitivity. Some autoimmune hemolytic anemias can be drug related, but the hemolysis is predominantly extravascular. (CH14)

A 32-year-old woman from Hanoi, Vietnam, gives birth at 34 weeks' gestation to a markedly hydropic stillborn male infant. Autopsy findings include hepatosplenomegaly and cardiomegaly, serous effusions in all body cavities, and generalized hydrops. No congenital anomalies are noted. There is marked extramedullary hematopoiesis in visceral organs. Which of the following hemoglobins is most likely predominant on hemoglobin electrophoresis of the fetal RBCs? A Hemoglobin A1 B Hemoglobin A2 C Hemoglobin Bart's D Hemoglobin E E Hemoglobin F F Hemoglobin H

C Hemoglobin Bart's The infant had α-thalassemia major, which is most likely to occur in individuals of Southeast Asian ancestry, each of whose parents could have two abnormal α-globin genes on chromosome 16. A complete lack of α-globin chains precludes formation of hemoglobins A1, A2, and F. Only a tetramer of γ chains (Bart's hemoglobin) can be made, leading to severe fetal anemia. Inheritance of three abnormal α-globin chains leads to hemoglobin H disease, with tetramers of β chains; survival to adulthood is possible. Hemoglobin E disease produces mild hemolytic anemias. (CH14)

A 60-year-old man has noted a nodule in his neck that has increased rapidly in size over the past 2 months. On physical examination, there is a firm, nontender, 10-cm mass in the left lateral posterior neck that appears to be fused cervical lymph nodes. Hepatosplenomegaly is noted. A head CT scan reveals a mass in the Waldeyer ring near the pharynx. A biopsy of the neck mass is performed, and on microscopic examination shows abnormal lymphoid cells with many mitotic figures and many apoptotic nuclei. He is treated with a cocktail of cell cycle-acting chemotherapeutic agents. The cervical and oral masses shrink dramatically over the next month. Based on his history and response to treatment, the tumor cells are most likely to have which of the following features? A Diminished vascularity B Evolution of polyclonality C High growth fraction D Limited capacity to metastasize E Strong expression of tumor antigens

C High growth fraction Some neoplasms, including certain lymphomas, have a high proportion of cells in the replicative pool (i.e., have high growth fraction). They grow rapidly and respond rapidly to drugs that kill dividing cells. Poor vascularity would not fa- vor rapid growth, and many neoplasms elaborate growth fac- tors that promote vascular proliferation. Monoclonality rather than polyclonality is typical of malignant tumors, though subclones of neoplastic cells do arise over time. Aggressive neoplasms tend to be more likely to metastasize. Tumors that are highly antigenic are likely to be controlled by the immune system and not to be rapidly growing. (CH7)

A 66-year-old man with chronic cough has an episode of hemoptysis. On physical examination, there are no abnormal findings. A chest radiograph shows a 6-cm mass in the right lung. A sputum cytologic analysis shows neoplastic squamous cells. Metastases from his lung lesion are most likely to be found at which of the following sites? A Cerebral hemisphere B Chest wall muscle C Hilar lymph nodes D Splenic red pulp E Vertebral bone marrow

C Hilar lymph nodes Carcinomas metastasize through lymphatics most of- ten, usually to regional nodes first. Hematogenous metastases are possible, however, to sites such as bone marrow, liver, or the opposite lung. About half of all cerebral metastases arise from lung primary carcinomas. Soft-tissue metastases to muscle, fat, and connective tissues are rare, as are splenic metastases. (CH7)

A 62-year-old man has had increasing knee pain with movement for the past 10 years. The knee joint surfaces are eroded and the joint space narrowed. There is loss of compressibility and lubrication of articular cartilaginous surfaces. Loss of which of the following extracellular matrix components has most likely occurred in this man? A Elastin B Fibronectin C Hyaluronan D Integrin E Laminin

C Hyaluronan He has osteoarthritis, or degenerative joint disease, with loss of articular hyaline cartilage. Hyaluronan (hyaluronic acid) is a large mucopolysaccharide, one form of proteoglycan, which forms a hydrated, compressible gel contributing to the shock-absorbing function of joint surfaces. Elastin is a fibrillar protein that provides recoil in tissues such as skin, arterial walls, and ligaments that need to stretch and return to their original shape. Fibronectin is a form of glycoprotein that serves an adhesive function. Integrins are glycoproteins that serve as cellular receptors for extracellular matrix components; they can link to intracellular actin so that cells can alter their shape and mobility. Laminins are a form of glycoprotein that help to anchor epithelial surfaces in basement membranes. (CH1)

A 44-year-old woman feels painless lumps in her armpit, which were not present a month ago. On examination, right axillary lymphadenopathy is present. The nodes are painless but firm. Which of the following is the most likely lesion in her right breast? A Acute mastitis with abscess B Fibroadenoma C Infiltrating lobular carcinoma D Intraductal carcinoma E Leiomyosarcoma

C Infiltrating lobular carcinoma Lymphatic spread, especially to regional lymph nodes draining from the primary site, is typical of a carcinoma. An intraductal carcinoma has not extended beyond the basement membrane, but an infiltrating carcinoma has acquired the ability to invade and spread via metastasis. The primary site may be difficult to detect if small or deep, and hence the need for radiologic imaging, such as mammography. A fibroadenoma is a benign neoplasm and cannot invade or metastasize. Infection from a breast abscess can spread to the lymph nodes, but the resulting nodal enlarge- ment is typically associated with pain—a cardinal sign of acute inflammation. Sarcomas uncommonly metastasize to lymph nodes, and a leiomyosarcoma of breast is rare. (CH7)

A 55-year-old man has had hemoptysis and worsening cough for the past month. On physical examination, wheezes are auscultated over the right lung posteriorly. A chest radiograph shows a 6-cm right perihilar mass. A fine-needle aspiration biopsy is performed and yields cells with the microscopic appearance of non-small cell bronchogenic carcinoma. Molecular analysis of the neoplastic cells shows a p53 gene mutation. Which of the following mechanisms has most likely produced the neoplastic transformation? A Inability to hydrolyze GTP B Growth factor receptor activation C Loss of cell cycle arrest D Microsatellite instability E Transcriptional activation

C Loss of cell cycle arrest A p53 mutation involving both wild type alleles is one of the most common genetic alterations in human cancers, including the most common cancers—lung, colon, and breast. The loss of this tumor suppressor activity indicates that the cell cycle is not properly arrested in the late G1 phase, and when DNA damage occurs, DNA repair cannot be completed before the cell proliferates. Inability to hydrolyze GTP is a result of RAS oncogene activation. Growth factors such as EGF are activators of the cell cycle to promote cell growth. Microsatellite instability occurs with mutation in genes, such as hMSH2, that repair DNA damage. Transcriptional activation is a feature of the MYC proto-oncogene. (CH7)

A 38-year-old woman has abdominal distention that has been worsening for the past 6 weeks. An abdominal CT scan shows bowel obstruction caused by a 6-cm mass in the jejunum. At laparotomy, a portion of the small bowel is resected. Flow cytometric analysis of a portion of the tumor shows a clonal population of B lymphocytes with high S phase. Translocation with activation of which of the following nuclear oncogenes is most likely to be present in this tumor? A APC B EGF C MYC D p53 E RAS

C MYC The MYC oncogene is commonly activated in Burkitt lymphoma because of a t(8;14) translocation. The MYC gene binds DNA to cause transcriptional activation of growth- related genes such as that for cyclin D1, resulting in activa- tion of the cell cycle. EGF (such as HER2 in breast cancers) encodes the epithelial growth factor receptor located on the cell surface. p53 and APC are tumor suppressor genes that are inactivated in many cancers, including colon cancer. RAS oncogene encodes a GTP-binding protein that is located un- der the cell membrane. (CH7)

A 29-year-old woman has had fatigue with dizziness for the past 5 months. On physical examination, she has an erythematous malar rash. She has no lymphadenopathy, but there is a palpable spleen tip. She is afebrile. Laboratory studies show hemoglobin, 8.9 g/dL; hematocrit, 27.8%; MCV, 103 μm3; RBC distribution width index, 22; WBC count, 8650/mm3; platelet count, 222,000/mm3; and reticulocyte count, 3.3%. The peripheral blood smear shows polychromasia, but no schistocytes. Her serum total bilirubin is 3.2 mg/dL with direct bilirubin 0.8 mg/dL, and haptoglobin is 5 mg/dL. Antinuclear antibody and anti-double-stranded DNA tests are positive. What additional laboratory test finding is she most likely to have? A D-dimer 10 μg/mL B Increased RBC osmotic fragility C Positive Coombs test D Serum cobalamin (vitamin B12) 50 pg/mL E Serum ferritin 240 ng/mL

C Positive Coombs test She has a circulating antibody against her RBCs leading to hemolytic anemia. The indirect antiglobulin (Coombs) test detects antibody in the plasma. The direct antiglobulin (Coombs) test detects antibody bound to RBCs. Autoimmune hemolytic anemias can be a feature of autoimmune diseases, such as systemic lupus erythematosus in this woman. Most of the hemolysis is extravascular in the spleen, but some can be intravascular. The reticulocyte count is typically increased (polychromasia) with hemolysis, and serum haptoglobin is diminished. An elevated D-dimer suggests a microangiopathic hemolytic anemia, but she has no schistocytes on the peripheral blood smear. Increased osmotic fragility is a feature of RBCs in paroxysmal nocturnal hemoglobinuria. Her mild macrocytosis indicates increased reticulocytosis, not vitamin B12 deficiency, and hemolysis is not part of pernicious anemia. Increased serum ferritin is typical for anemia of chronic disease (mild increase) or hemochromatosis (marked increase). (CH14)

A proponent of Chilean Malbec, Syrah, and Merlot wines (all reds) touts their contribution to longevity, but this wine aficionado also controls his dietary caloric content so that his body mass index is <22. This lifestyle promotes increased insulin sensitivity and glucose utilization. He fully expects to live longer because he has read that caloric restriction pro- longs life. In this man, which of the following intracellular substances will most likely mediate the effect of calorie restriction upon increased longevity? A Caspase B Glutathione C Sirtuins D Telomerase E Ubiquitin

C Sirtuins The one sure way to increase life span is calorie re- striction. But why do without the things we like, only to do without them longer? Dietary excesses lead to increased mor- bidity with reduced quality of life, as well as mortality, from chronic diseases such as diabetes mellitus. The activity of sir- tuins on histone acetylation and deacetylation may promote transcription of genes encoding for proteins that increase metabolic activity and inhibit effects of free radicals. Red wines have been shown to increase sirtuins, but don't drink too much! Moderation is the key. Glutathione promotes free radical breakdown, although chronic excessive alcohol con- sumption depletes hepatocyte glutathione. Caspases trigger apoptosis and cell death. Telomerases aid in promoting con- tinued cell division, but cannot be altered by lifestyle, and turning them on is one feature of neoplasia. Ubiquitin is a peptide that is part of the ubiquitin-proteasome pathway of protein degradation seen with nutrient deficiencies, so when you eat less, be sure to eat a balanced diet. (CH1)

Since childhood, a 30-year-old man has been easily fatigued with minimal exercise. Laboratory studies show hypochromic microcytic anemia. Hemoglobin electrophoresis reveals decreased Hgb A1 with increased Hgb A2 and Hgb F. His serum ferritin is markedly increased. Which of the following mutations is most likely to be present in the β-globin gene of this man? A New stop codon B Single base insertion, with frameshift C Splice site D Three-base deletion E Trinucleotide repeat

C Splice site This is one mechanism for β+ thalassemia. Because the introns are usually involved, the flanking exons remain, and some normal splicing can occur, so that some β-globin chain synthesis can occur, but not sufficient for adequate hemoglobin production. The other listed mutations lead to a block in translation, with no functional β-globin chain synthesis, typical for β0 thalassemia. (CH14)

A 40-year-old man has a history of intravenous drug use. Physical examination shows needle tracks in his left antecubital fossa. He has mild scleral icterus. Serologic studies for HBsAg and anti-HCV are positive. He develops hepatocellular carcinoma 15 years later. Which of the following viral characteristics best explains why this patient developed hepatocellular carcinoma? A Viral integration in the vicinity of proto-oncogenes B Viral capture of proto-oncogenes from host cellular DNA C Viral inflammatory changes with genomic damage D Viral inactivation of RB and p53 gene expression E Viral infection of inflammatory cells with host immunosuppression

C Viral inflammatory changes with genomic damage Although the hepatitis B virus (HBV) and hepatitis C virus (HCV) genomes do not encode for any transforming proteins, the regenerating hepatocytes are more likely to develop mutations, such as inactivation of p53. HBV does not have a consistent site of integration in the liver cell nuclei, and it does not contain viral oncogenes. Many DNA viruses, such as human papillomavirus (HPV), inactivate tumor suppressor genes, but there is no convincing evidence that HBV or HCV can bind to p53 or RB proteins. Also, the HBV-encoded regulatory element, called HBx, disrupts normal growth of infected hepatocytes. Neither HBV nor HCV infects immune cells. (CH7)

A 25-year-old woman has a 3-year history of arthralgias. Physical examination shows no joint deformity, but she appears pale. Laboratory studies show total RBC count of 4.7 million/mm3, hemoglobin of 12.5 g/dL, hematocrit of 37.1%, platelet count of 217,000/mm3, and WBC count of 5890/mm3. The peripheral blood smear shows hypochromic and microcytic RBCs. Total serum iron and ferritin levels are normal. Hemoglobin electrophoresis shows 93% hemoglobin A1 with elevated hemoglobin A2 level of 5.8% and hemoglobin F level of 1.2%. What is the most likely diagnosis? A Anemia of chronic disease B Autoimmune hemolytic anemia C β-Thalassemia minor D Infection with Plasmodium vivax E Iron deficiency anemia

C β-Thalassemia minor Although β-thalassemia minor and iron deficiency anemia are both characterized by hypochromic and microcytic RBCs, there is no increase in hemoglobin A2 in iron deficiency states. A normal serum ferritin level also excludes iron deficiency. In contrast to β-thalassemia major, there is usually a mild anemia without major organ dysfunction with β-thalassemia minor. Diseases that produce hemolysis and increase erythropoiesis (e.g., autoimmune hemolytic anemia, malaria) do not alter the composition of β-globin chain production. Anemia of chronic disease may mimic iron deficiency and thalassemia minor with respect to hypochromia and microcytosis; however, anemia of chronic disease is associated with an increase in the serum concentration of ferritin. (CH14)

In a clinical trial, patients diagnosed with malignant melanoma are treated by infusion of autologous CD8+ T cells grown in vitro. These CD8+ T cells are known to kill melanoma cells, but not normal cells. Which of the following target antigens in the tumor cells are most likely recognized by these CD8+ T cells? A Class I MHC molecules with a melanoma cell peptide B Class I MHC molecules with a peptide from normal melanocytes and melanoma cells C Class I MHC molecules plus a peptide derived from carcinoembryonic antigen D Class II MHC molecules with a melanoma cell peptide E Class II MHC molecules with a peptide from normal melanocytes and melanoma cells F Class II MHC molecules with laminin receptors on melanoma cells

Class I MHC molecules with a melanoma cell peptide All human nucleated cells express MHC class I antigens. CD8+ T cells recognize peptides presented by MHC class I antigens. In many tumors, especially melanomas, the tumor cells produce peptides that can be presented by MHC class I molecules. Such tumor-specific peptides are not produced by other cells, so the CD8+ T cells specific for such peptides lyse melanoma cells, but not normal melanocytes or other normal cells. (CH7)

A study of patients treated with chemotherapy protocols for cancer shows that 10% of them subsequently develop a second cancer, a much higher incidence compared with a control group not receiving chemotherapy. These chemotherapy protocols included the alkylating agent cyclophosphamide. What is the most likely mechanism by which this agent causes carcinogenesis in these treated cancer patients? A Activation of protein kinase C B Activation of endogenous viruses C Blockage of TGF-β pathways D Direct DNA damage E Inhibition of DNA repair F Inhibition of telomerase

D Direct DNA damage Chemical carcinogens can have highly reactive electrophilic groups that can directly damage DNA, leading to mutations. Direct-acting agents, such as alkylating chemotherapy drugs, do not require conversion to a carcinogen. Some environmental toxic agents, such as polycyclic hydrocarbons, require metabolic conversion to a carcinogen and are called indirect-acting agents. Phorbol esters are examples of promoters of chemical carcinogenesis that cause tumor promotion by activating protein kinase C. This enzyme phosphorylates several substrates in signal transduction pathways, including those activated by growth factors, and the cells divide. Forced cell division predisposes the accumulation of mutations in cells previously damaged by exposure to a mutagenic agent (initiator). The TGF-β pathways work via growth inhibition. Proteins such as p53 that function in DNA repair pathways can become nonfunctional through mutation. Viral infections such as hepatitis B and C tend to promote growth by binding to p53 and inactivating its protective function. Telomerase activity is not affected by carcinogens. (CH7)

A 65-year-old man diagnosed with follicular non-Hodgkin lymphoma is treated with chemotherapy. He develops fever and cough of a week's duration. On examination, there are bilateral pulmonary rales. A chest radiograph shows diffuse interstitial infiltrates. A sputum specimen is positive for cytomegalovirus. He develops scleral icterus and Raynaud phenomenon. Laboratory studies show hemoglobin, 10.3 g/dL; hematocrit, 41.3%; MCV, 101 μm3; WBC count, 7600/mm3; and platelet count, 205,000/ mm3. His serum total bilirubin is 6 mg/dL, direct bilirubin is 0.8 mg/dL, and LDH is 1020 U/L. Coombs test is positive. Which of the following is the most likely mechanism for his anemia? A Marrow aplasia caused by chemotherapy B Vitamin K deficiency caused by cytomegalovirus hepatitis C Megaloblastic anemia caused by folate deficiency D Extravascular hemolysis caused by cold agglutinins E Iron deficiency caused by metastases to colon

D Extravascular hemolysis caused by cold agglutinins The findings point to Coombs-positive immune he- molytic anemia with indirect hyperbilirubinemia. Cold agglu- tinin immunohemolytic anemia can be seen with lymphoid neoplasms and infections such as Mycoplasma, Epstein-Barr virus, HIV, influenza virus, and cytomegalovirus (CMV). IgM binds to RBCs at cooler peripheral body regions and then fixes complement. At warmer central regions, the antibody is eluted, but the complement marks the RBCs for extravascular destruction in the spleen, but there is minimal intravascular hemolysis. The increased RBC turnover increases the MCV and the bilirubin, which is mainly indirect. Chemotherapy can suppress bone marrow production, but more likely all cell lines, and without an immune component. Although this patient has CMV infection, CMV hepatitis would likely in- crease direct and indirect bilirubin, and not account for ane- mia. Folate deficiency could account for macrocytosis, but not a positive Coombs test. Non-Hodgkin lymphomas do not of- ten involve colon, but this might account for gastrointestinal bleeding with features of iron deficiency and microcytosis.

A 76-year-old woman has reported a change in the caliber of her stools during the past month. On physical examination, there are no abnormal findings, but a stool sample is positive for occult blood. A colonoscopy is performed and a constricting mass involving the lower sigmoid colon is found. She undergoes a partial colectomy. Which of the following techniques used during surgery can best aid the surgeon in determining whether the resection is adequate to reduce the probability of a recurrence? A Electron microscopy B Fine-needle aspiration C Flow cytometry D Frozen section E Radiologic imaging F Serum carcinoembryonic antigen assay

D Frozen section The rapid frozen section of resection margins helps to determine whether enough of the colon has been resected. Electron microscopy requires at least 1 day to perform, and helps to determine the cell type, but it has largely been sup- planted by immunohistochemistry. Fine-needle aspiration is used for preoperative diagnosis to find a malignancy. Flow cytometry can be performed in several hours, but it is useful mainly for prognostic information and is not a "stat" procedure. Radiologic imaging aids in preoperative diagnosis and assessment of possible metastases. Serum tumor markers may aid in preoperative diagnosis or postoperative follow-up of neoplasms. (CH7)

An experiment analyzes factors involved in the cell cycle during growth factor-induced cellular regeneration in a tissue culture. Cyclin B synthesis is induced; the cyclin B binds and activates cyclin-dependent kinase 1 (CDK1). The active kinase produced by this process is most likely to control progression in which of the following phases of the cell cycle? A G0 to G1 B G1 to S C S to G2 D G2 to M E M to G1

D G2 to M CDK1 controls an extremely important transition point, the G2 to M transition, during the cell cycle, which can be regulated by CDK inhibitors. The other checkpoints listed are regulated by a distinct set of proteins. (CH1)

A 50-year-old woman has had easy fatigability and noted a dragging sensation in her abdomen for the past 5 months. Physical examination reveals that she is afebrile. She has marked splenomegaly, but no lymphadenopathy. Laboratory studies show her total WBC count is 250,000/mm3 with WBC differential count showing 64% segmented neutrophils, 11% band neutrophils, 7% metamyelocytes, 5% myelocytes, 4% myeloblasts, 3% lymphocytes, 2% basophils, 2% eosinophils, and 2% monocytes. A bone marrow biopsy is performed, and karyotypic analysis of the cells reveals a t(9;22) translocation. Medical treatment with a drug having which of the following modes of action is most likely to produce a complete remission in this patient? A Activating cellular caspases B Antibody binding to EGF receptors C Delivering normal p53 into cells with viral vectors D Inhibiting tyrosine kinase activity E Preventing translocation of β-catenin to the nucleus

D Inhibiting tyrosine kinase activity Thispatienthasaclassichistoryandt(9;22)transloca- tion with chronic myelogenous leukemia. The translocation causes uncontrolled nonreceptor tyrosine kinase activity of the BCR-ABL fusion gene. These patients undergo remission with drugs such as imatinib that inhibit tyrosine kinases. Agents that activate caspases theoretically may help in many cases, especially when apoptosis is blocked as in tumors with BCL2 overexpression. Antibodies to epithelial growth factor receptors, such as ERBB2 (HER2) receptors, are beneficial in certain breast tumors with amplification of this gene. Deliv- ery of p53 into cells by viral vectors has not yet been proven to be valuable in cancer treatment, and it is not used in chronic myelogenous leukemia. Translocation of β-catenin to the nucleus occurs in colon cancers when there is mutational loss of APC genes. (CH7)

A 10-year-old child has experienced multiple episodes of pneumonia and meningitis with septicemia since infancy. Causative organisms include Streptococcus pneumoniae and Haemophilus influenzae. On physical examination, the child has no organomegaly and no deformities. Laboratory studies show hemoglobin of 9.2 g/dL, hematocrit of 27.8%, platelet count of 372,000/mm3, and WBC count of 10,300/mm3. A hemoglobin electrophoresis shows 1% hemoglobin A2, 7% hemoglobin F, and 92% hemoglobin S. Which of the following is the most likely cause of the repeated infections in this child? A Absent endothelial cell expression of adhesion molecules B Diminished hepatic synthesis of complement proteins C Impaired neutrophil production D Loss of normal splenic function E Reduced synthesis of immunoglobulins

D Loss of normal splenic function In sickle cell anemia, the cumulative ischemic dam- age to the spleen results in autosplenectomy, leaving behind a small fibrotic remnant of this organ. The impaired splenic function and resultant inability to clear bacteria from the bloodstream can occur early in childhood, leading to risk for infection with encapsulated bacterial organisms. Immunodeficiency results from lack of splenic function, not from lack of immunoglobulins. Endothelium can be damaged with sickling, and adhesion between endothelial cells and RBCs is increased in sickle cell anemia. Complement proteins are part of innate immune responses in acute inflammation. There is no impairment in production or function of neutrophils. (CH14)

A 40-year-old woman has had chronic congestive heart failure for the past 3 years. In the past 2 months, she developed a cough productive of rust-colored sputum. A sputum cytology specimen now shows numerous hemosiderin-laden macrophages. Which of the following subcellular structures in these macrophages is most important for the accumulation of this pigment? A Chromosome B Endoplasmic reticulum C Golgi apparatus D Lysosome E Ribosome

D Lysosome Heterophagocytosis by macrophages requires that endocytosed vacuoles fuse with lysosomes to degrade the engulfed material. With congestive heart failure, extravasation of RBCs into alveoli occurs, and pulmonary macrophages must phagocytose the RBCs, breaking down the hemoglo- bin and recycling the iron by hemosiderin formation. The other listed options are components that play a role in cell synthetic functions. (CH1)

A 69-year-old woman has experienced increasing mal- aise and a 10-kg weight loss over the past year. She dies of massive pulmonary thromboembolism. The gross appearance of the liver at autopsy is shown in the figure. Which of the fol- lowing best describes the lesions seen in her liver? A Invasive angiosarcoma B Hepatocellular carcinoma C Leukemic infiltration D Metastatic adenocarcinoma E Multifocal hepatic adenomas

D Metastatic adenocarcinoma The figure shows the appearance of multiple variably sized tan metastatic lesions in the liver from hematogenous spread of carcinoma. Adenocarcinomas from abdominal primary sites such as colon, pancreas, and stomach are most likely. Thromboembolism suggests a hypercoagulable state such as a paraneoplastic syndrome. Angiosarcomas of the liver are uncommon. A primary malignancy typically appears as a dominant mass, not multiple masses. Although some benign tumors, such as leiomyomas of the uterus, can be multiple, this is not the rule in the liver, and hepatic adenomas are rare. Although hepatocellular carcinomas can have "satellite" nodules, widespread nodules such as those seen in the figure are more characteristic of metastases. Leukemic infiltrates typically do not produce large mass lesions, though some lymphomas may do so. Resection of multiple metastases is usually futile. (CH7)

A 50-year-old woman undergoes screening colonoscopy as part of a routine health maintenance work-up. An isolated 1-cm pedunculated polyp is found in the sigmoid colon. The excised polyp histologically shows well-differentiated glands with no invasion of the stalk. Which of the following investigational research procedures can distinguish most clearly whether the polyp represents hyperplasia of the colonic mucosa or a tubular adenoma? A Flow cytometry to quantitate cells in the S phase B Histochemical staining for mucin C Immunohistochemical staining for keratin D Molecular marker of clonality

D Molecular marker of clonality A true neoplasm is a monoclonal proliferation of cells, whereas a reactive proliferation of cells is not monoclonal. Mo- lecular genetic analysis, such as allelotype analysis with mic- rosatellite markers, shows clonality. Reactive and neoplastic cellular proliferations may have similar histochemical and immunohistochemical staining patterns based on the type of cells and proteins that are present. Flow cytometry is effective at indicating the DNA content, aneuploidy, and growth frac- tion, but does not indicate clonality. (CH7)

A 16-year-old boy notes passage of dark urine. He has a history of multiple bacterial infections and venous thromboses for the past 10 years, including portal vein thrombosis in the previous year. On physical examination, his right leg is swollen and tender. CBC shows hemoglobin, 9.8 g/dL; hematocrit, 29.9%; MCV, 92 μm3; platelet count, 150,000/mm3; and WBC count, 3800/mm3 with 24% segmented neutrophils, 1% bands, 64% lymphocytes, 10% monocytes, and 1% eosinophils. He has a reticulocytosis, and his serum haptoglobin level is very low. A mutation affecting which of the following gene products is most likely to give rise to this clinical condition? A β-Globin chain B Factor V C Glucose-6-phosphate dehydrogenase D Phosphatidylinositol glycan A (PIGA) E Prothrombin G20210A F Spectrin

D Phosphatidylinositol glycan A (PIGA) Paroxysmal nocturnal hemoglobinuria (PNH) is a disorder that results from an acquired stem cell membrane defect produced by a PIGA gene mutation that prevents the membrane expression of certain proteins that require a gly- colipid anchor. These include proteins that protect cells from lysis by spontaneously activated complement. As a result, RBCs, granulocytes, and platelets are exquisitely sensitive to the lytic activity of complement. The RBC lysis is intra- vascular, so patients can have hemoglobinuria (dark urine). Defects in platelet function are believed to be responsible for venous thrombosis. Recurrent infections can be caused by impaired leukocyte functions. Patients with PNH may develop acute leukemia or aplastic anemia as complications. Mutations in the β-globin chain can give rise to hemoglobinopathies such as sickle cell anemia. Patients with factor V (Leiden) and prothrombin G20210A mutations can present with thromboses, but there is no anemia or leukopenia. Patients with glucose-6-phosphate dehydrogenase (G6PD) deficiency have an episodic course from exposure to agents such as drugs that induce hemolysis. Spectrin mutations give rise to hereditary spherocytosis. (CH14)

In an epidemiologic study of anemias, the findings show that there is an increased prevalence of anemia in individuals of West African ancestry. By hemoglobin electrophoresis, some individuals within this region have increased hemoglobin S levels. The same regions also have a high prevalence of an infectious disease. Which of the following infectious agents is most likely to be endemic in the region where such anemia shows increased prevalence? A Borrelia burgdorferi B Clostridium perfringens C Cryptococcus neoformans D Plasmodium falciparum E Treponema pallidum F Trypanosoma brucei

D Plasmodium falciparum Throughout human history, malaria has influenced the increasing gene frequency of hemoglobin S. Individuals who are heterozygous for hemoglobin S have the sickle cell trait. They are more resistant to malaria, particularly the most malignant form caused by P. falciparum, because the parasites grow poorly or die at low oxygen concentrations, perhaps because of low potassium levels caused by potas- sium efflux from RBCs on hemoglobin sickling. The malarial parasite has difficulty completing its life cycle, even in cells with moderate amounts of hemoglobin S, as found in heterozygotes, giving a selective advantage to such persons living in endemic areas for falciparum malaria. Borrelia burgdorferi is the spirochete that causes Lyme disease. Clostridium perfringens may produce gas gangrene after soft-tissue injuries. Cryptococcus neoformans can cause granulomatous disease in immunocompromised individuals. Treponema pallidum is the infectious agent causing syphilis. Trypanosoma brucei infection causes sleeping sickness. (CH14)

Dermal fibroblasts are harvested from the skin biopsy specimen of an adult man. These fibroblasts are transduced with genes encoding for transcription factors including SOX2 and MYC. Under appropriate culture conditions these cells are then able to generate endodermal, mesodermal, and ectodermal cells. Into which of the following kinds of stem cell have these fibroblasts been transformed? A Embryonic B Lineage-committed C Mesenchymal D Pleuripotent

D Pleuripotent These transformed cells are designated iPS cells because they have been induced to become pleuripotent. This transformation process gets around the problem of using embryonic stem (ES) cells derived from manipulation of human embryos, which raises ethical and religious concerns. Embryonic stem cells are totipotent, but they become pleuripotent cells that can further divide into many different cell lines, yet maintain themselves in a replicating pool. Thus pleuripotent cells are the next best thing compared to embryonic cells for deriving human cells that could replace damaged or diseased tissues. Further differentiation of pleuripotent cells gives rise to cells with more restricted developmental capacity, such as mesenchymal stem cells that can give rise to tissues such as muscle and cartilage but not to endodermal or ectodermal cells. (CH1)

A 56-year-old woman has had vaginal bleeding for 1 week. Her last menstrual period was 10 years ago. On physical examination, a lower abdominal mass is palpated. An endometrial biopsy is performed and shows endometrial carcinoma. An abdominal CT scan shows a 6-cm mass in the left ovary. A total abdominal hysterectomy is performed. Microscopically, the ovarian mass is a granulosatheca cell tumor producing estrogen. Which of the following best describes the relationship between these two neoplasms? A Genetic susceptibility to tumorigenesis B Mutational inactivation of a tumor suppressor gene C Paraneoplastic syndrome D Promotion of carcinogenesis E Tumor heterogeneity

D Promotion of carcinogenesis Estrogen, similar to many other hormones and drugs, by itself is not carcinogenic, but it is responsible for stimulation of endometrial growth (hyperplasia), which has a promoting effect when cellular mutations occur to produce carcinoma. Inherited susceptibility can never be completely excluded when an individual has two tumors; this can occur in patients with inherited mutations in the p53 gene. In this case, however, there is a clear hormonal basis for the second tumor. Faulty tumor suppressor genes are not involved in hormonal promotion of a neoplasm. A paraneoplastic syndrome results from ectopic secretion of a hormone by tumor (e.g., lung cancer cells producing corticotropin). Tumor heterogeneity does not refer to two separate kinds of neoplasms; it refers to heterogeneity with a given tumor or metastasis. (CH7)

A 70-year-old woman reported a 4-month history of a 4-kg weight loss and increasing generalized icterus. On physical examination, she has midepigastric tenderness on palpation. An abdominal CT scan shows a 5-cm mass in the head of the pancreas. Fine-needle aspiration of the mass is performed. On biochemical analysis, the neoplastic cells show continued activation of cytoplasmic kinases. Which of the following genes is most likely to be involved in this process? A APC B MYC C p53 D RAS E RET F sis

D RAS The RAS oncogene is the most common oncogene involved in the development of human cancers. Mutations of the RAS oncogene reduce GTPase activity, and RAS is trapped in an activated GTP-bound state. RAS then signals the nucleus through cytoplasmic kinases. The APC gene can cause activation of the WNT signaling pathway. The MYC oncogene is a transcriptional activator that is overex- pressed in many tumors. The p53 tumor suppressor gene encodes a protein involved in cell cycle control. The RET proto-oncogene encodes a receptor tyrosine kinase involved in neuroendocrine cells of the thyroid, adrenal medulla, and parathyroids. The sis oncogene encodes platelet-derived growth factor receptor-β, which is overexpressed in certain astrocytomas. (CH7)

An 18-year-old woman from Copenhagen, Denmark, has had malaise and a low-grade fever for the past week, along with arthralgias. On physical examination, she appears very pale, except for a bright red malar facial rash. She has a history of chronic anemia, and spherocytes are observed on a peripheral blood smear. Her hematocrit, which normally ranges from 35% to 38%, is now 28%, and the reticulocyte count is very low. The serum bilirubin level is 0.9 mg/dL. Which of the following events is most likely to have occurred in this patient? A Accelerated extravascular hemolysis in the spleen B Development of anti-RBC antibodies C Disseminated intravascular coagulation D Reduced erythropoiesis from parvovirus infection E Superimposed dietary iron deficiency

D Reduced erythropoiesis from parvovirus infection Patients with hereditary spherocytosis may have an aplastic crisis precipitated by a parvovirus infection. In adults who do not have a defect in normal RBC production, such as hereditary spherocytosis or sickle cell anemia, or who are not immunosuppressed, parvovirus infection is self- limited and often goes unnoticed. When there is an under- lying RBC production defect, then RBC production is shut down by parvovirus, and there is no reticulocytosis. Dissem- inated intravascular coagulation gives rise to thrombocyto- penia, bleeding, and the appearance of fragmented RBCs in the blood smear. Reticulocytosis would be prominent with hemolysis and with RBC antibodies. Iron deficiency does not occur in hemolytic anemias because the iron that is released from hemolyzed cells is reused. (CH14)

A 53-year-old woman has noticed increasing malaise. On physical examination, there are no abnormal findings, but a stool guaiac test is positive. Her hemoglobin level is 7.9 g/dL. A colonoscopy is performed, and a 3-cm sessile mass is found in the cecum. A biopsy specimen of the mass shows a moderately differentiated adenocarcinoma confined to the mucosa. An abdominal CT scan shows no lymphadenopathy or hepatic lesions. Given this information, which of the following is the best course of action? A Administer a multiagent chemotherapeutic regimen B Observe the lesion for further increase in size C Remove the entire colon to prevent a recurrence D Resect the tumor and some normal surrounding E Search for a primary malignancy in another organ

D Resect the tumor and some normal surrounding A malignant epithelial neoplasm arises in the mucosa but has a tendency to invade locally. A benign neoplasm is often well circumscribed, and compressed normal surround- ing tissue appears to form a discrete border. This localized le- sion can be resected easily, with adequate margins. Without evidence for spread outside the colon, chemotherapy is un- likely to be of benefit. The biopsy specimen shows a malig- nant lesion; it must be removed before it increases in size and invades locally or metastasizes. If there is no family history, a familial cancer with high risk of recurrence from multiple polyps is unlikely; local excision is adequate. Such a solitary mucosal lesion is unlikely to represent a metastasis. (CH7)

A healthy 19-year-old woman suffered blunt abdominal trauma in a motor vehicle accident. On admission to the hospital, her initial hematocrit was 33%, but over the next hour, it decreased to 28%. A paracentesis yielded serosanguineous fluid. She was taken to surgery, where a liver laceration was repaired, and 1 L of bloody fluid was removed from the peritoneal cavity. She remained stable. A CBC performed 3 days later is most likely to show which of the following morphologic findings in the peripheral blood? A Basophilic stippling of red cells B Hypochromic red cells C Leukoerythroblastosis D Reticulocytosis E Schistocytosis

D Reticulocytosis The acute blood loss, in this case intraperitoneal hemor- rhage, results in a reticulocytosis from marrow stimulation by anemia. Basophilic stippling of RBCs suggests a marrow injury, such as with a drug or toxin. Hypochromic RBCs occur in iron deficiency and thalassemias, both associated with reduced hemoglobin synthesis. Acute blood loss does not give rise to iron deficiency if iron stores and diet are adequate. Leukoerythroblastosis is typical of a myelophthisic process in the marrow, with both immature WBCs (myelocytes) and RBCs (nucleated forms) present. Schistocytes suggest a microangiopathic hemolytic anemia, which can accompany shock or sepsis. (CH14)

A 66-year-old woman has worked all of her life on a small family farm on the Kantō Plain near Tokyo. She has had no previous major illnesses, but has been feeling increasingly tired and weak for the past year. On physical examination, she is afebrile, but appears pale. Laboratory studies show hemoglo- bin, 11.3 g/dL; hematocrit, 33.8%; platelet count, 205,200/mm3; and WBC count, 64,000/mm3. Immunophenotyping of her leukocytes yields the findings shown in the figure. Assum- ing that the dominant cell population is clonal, which of the following microbial agents is most likely involved in this patient's disease process? A Epstein-Barr virus B Hepatitis B virus C HIV-1 D Helicobacter pylori E Human T cell lymphotropic virus type 1

E Human T cell lymphotropic virus type 1 The largest cell population in the figure, determined to be clonal, is marking for CD4. This patient has a T cell leu- kemia, which develops in approximately 1% of individuals infected with human T cell lymphotropic virus type 1. In- fection with Epstein-Barr virus is associated with various cancers, including Burkitt lymphoma and nasopharyngeal carcinoma. Infection with hepatitis B virus may result in he- patic cirrhosis, in which hepatocellular carcinoma may arise. HIV-1 infection causes AIDS, with a diminished CD4+ cell count. H. pylori promotes chronic gastritis with increased risk for gastric adenocarcinomas and B cell lymphomas. (CH7)

A 59-year-old man has noticed blood in his urine for the past week. Cystoscopy shows a 4-cm exophytic mass involving the right bladder mucosa near the trigone. After biopsy specimens are obtained, he undergoes a radical cystectomy. Examination of the excised specimen shows an anaplastic carcinoma that has infiltrated the bladder wall. Which of the following techniques applied to the cells from his neoplasm is most likely to categorize the cell of origin? A Chromosomal karyotyping B Cytologic smear C DNA microarray D Flow cytometric analysis E Immunohistochemistry

E Immunohistochemistry Histologic sections from malignant neoplasms are frequently assessed with a panel of immunostains to detect antigenic characteristics, such as protein expression, to aid in characterizing the cell of origin, as well as provide infor- mation in selection of treatment protocols. In this case, the immunostains are likely to reveal that this neoplasm is a high-grade urothelial carcinoma. A cytology smear shows light microscopic findings helpful to screen for malignancy, but the findings often fall short of diagnosing a specific cell type. The other listed techniques are best for determination of treatment and prognosis. (CH7)

A 61-year-old woman has noted a feeling of pelvic heaviness for the past 6 months. On physical examination, there is a palpable nontender lower abdominal mass. An abdominal ultrasound scan shows a 12-cm solid mass in the uterine wall. A total abdominal hysterectomy is performed. The mass has the microscopic appearance of a well-differentiated leiomyosarcoma. One year later, a chest radiograph shows a 4-cm nodule in her right lower lung. Cytologic analysis of a fine-needle biopsy specimen of the nodule shows a poorly differentiated sarcoma. The patient's medical history indicates that she has smoked cigarettes most of her adult life. Which of the following mechanisms best explains these findings? A Continued cigarette smoking by the patient B Development of a second primary neoplasm C Inheritance of a defective RB gene D Immunodeficiency with HIV infection E Metastasis from an aggressive tumor subclone

E Metastasis from an aggressive tumor subclone Although neoplasms begin as monoclonal prolifera- tions, additional mutations occur over time, leading to subclones of neoplastic cells with various aggressive properties. This subcloning may allow metastases, greater invasiveness, resistance to chemotherapy, and morphologic differences to occur. Because sarcomas of the lung are rare, the lung mass is statistically a metastasis. Though second primary malignancies do arise, particularly in persons who have already had a malignancy, the odds favor a metastasis in a person with a prior malignancy. Sarcomas are not related to smoking tobac- co. Inheritance of a mutant RB gene is most likely to lead to childhood retinoblastomas and osteosarcomas. Kaposi sarco- ma is the sarcoma most often associated with AIDS from HIV infection. (CH7)

Various soluble mediators are added to a cell culture containing epidermal cells to determine which of the mediators might be useful for promoting epidermal cell growth. When epidermal growth factor (EGF) is added, it binds to epidermal cell surface receptors, with subsequent transcription factor translocation and DNA transcription. This effect in the epidermal cells is most likely to be mediated through which of the following intracellular pathways? A Calcium ion channel B Cyclic AMP C Cyclin-dependent kinase D JAK/STAT system E Mitogen-activated protein (MAP) kinase

E Mitogen-activated protein (MAP) kinase The MAP kinase cascade is involved in signaling from activation via cell surface receptors for growth factors. This pathway is particularly important for signaling of EGF and fibroblast growth factor. Ligand binding, such as occurs with acetylcholine at a nerve-muscle junction, alters the conformation of ion channel receptors to allow flow of spe- cific ions such as calcium into the cell, changing the electric potential across the cell membrane. Cyclic AMP is a second messenger that is typically activated via ligand binding to receptors with seven transmembrane segments that associ- ate with GTP-hydrolyzing proteins; chemokine receptors function in this fashion. Cyclin-dependent kinases act within the nucleus. JAK/STAT pathways typically are recruited by cytokine receptors. (CH1)

An experiment involving carcinoma cells grown in culture studies the anti-tumor surveillance effects of the innate immune system. These carcinoma cells fail to express MHC class I antigens. It is observed, however, that carcinoma cells are lysed when an immune cell that has been activated by IL-2 is added to the culture. Which of the following immune cells is most likely to function in this manner? A CD4+ lymphocyte B CD8+ lymphocyte C Macrophage D Neutrophil E NK cell F Plasma cell

E NK cell Several types of immune cells can recognize and help destroy tumor cells. Tumor antigens that are dis- played via MHC class I molecules can be recognized by cytotoxic CD8+ cells. Normal human cells should display MHC class I antigens, but many cancers do not display their antigens well, and when MHC class I molecules are not displayed, NK cells are triggered to target these cells for lysis. Macrophages may work in concert with CD8+ cells and NK cells to phagocytize and kill tumor cells when up-regulated by interferon-γ. CD4+ "helper" cells do not play a direct anticancer role. Neutrophils are ineffective against cancer cells, but may be attracted to areas of tumor necrosis. Plasma cells may produce antibodies directed against tumor antigens, but such antibodies are ineffective in controlling tumors. (CH7)

A 39-year-old woman underwent a routine health maintenance examination for the first time in many years. A Pap smear was obtained, and the result reported was abnormal. On pelvic examination, a red, slightly raised, 1-cm lesion on the anterior ectocervix at the 2 o'clock position was excised and biopsied. The microscopic appearance on medium-power magnification is shown in the figure. Which of the following is most characteristic of this patient's condition? A Primary neoplasm in the endometrium B Elevated CA-125 level in the serum C Positive HSV-2 molecular test in the lesion D Pulmonary nodules on a chest radiograph E No recurrence following local excision

E No recurrence following local excision The figure shows an in situ carcinoma of the squamous cervical epithelium with neoplastic growth only above the basement membrane. Tissue damage with repair and regen- eration may give rise to metaplasia, which may progress to dysplasia, considered premalignant. In situ cancers, limited to the epithelium, are noninvasive, and local excision has a 100% cure rate. In situ lesions do not give rise to metastases and have not arisen elsewhere. This lesion is related to hu- man papillomavirus (HPV) infection, not herpes simplex vi- rus (HSV). CA-125 is most often a tumor marker for ovarian cancer. (CH7)

A 30-year-old man has a 15-year history of increasing numbers of benign skin nodules. On physical examination, the firm, nontender, subcutaneous nodules average 0.5 to 1 cm. Further examination shows numerous oval 1- to 5-cm flat, light brown skin macules. Ophthalmoscopic examination shows hamartomatous nodules on the iris. A biopsy specimen of one skin nodule shows that it is attached to a peripheral nerve. Which of the following molecular abnormalities is most likely related to his clinical presentation? A Decreased susceptibility to apoptosis B Impaired functioning of mismatch repair C Increased production of epidermal growth factor D Lack of nucleotide excision repair E Persistent activation of the RAS gene F Reduced expression of RB protein

E Persistent activation of the RAS gene This patient has clinical features of neurofibromatosis type 1. The NF1 gene encodes for a GTPase-activating protein that facilitates the conversion of active (GTP-bound) RAS to inactive (GDP-bound) RAS. Loss of NF1 prevents such con- version and traps RAS in the active signal-transmitting stage that drives cell proliferation. Thus, the wild type NF1 gene acts as a tumor suppressor. All other listed mechanisms also are involved in carcinogenesis, but in different tumor types. (CH7)

In an experiment, surgical incisions are made in a study group of laboratory rats. Observations about the wounds are recorded over a 2-week period using various chemical mediators. Which of the following steps in the inflammatory-repair response is most likely affected by neutralization of transforming growth factor β (TGF-β)? A Chemotaxis of lymphocytes B Increase in vascular permeability C Leukocyte extravasation D Migration of epithelial cells E Production of collagen

E Production of collagen TGF-β stimulates many steps in fibrogenesis, includ- ing fibroblast chemotaxis and production of collagen by fi- broblasts, while inhibiting degradation of collagen. All of the other steps listed are unaffected by TGF-β. (CH1)

A 12-year-old boy has a history of episodes of severe abdominal, chest, and back pain since early childhood. On physical examination, he is afebrile, and there is no organomegaly. Laboratory studies show hemoglobin of 11.2 g/dL, platelet count of 194,000/mm3, and WBC count of 9020/mm3. The peripheral blood smear shows occasional sickled cells, nucleated RBCs, and Howell-Jolly bodies. Hemoglobin electrophoresis shows 1% hemoglobin A2, 6% hemoglobin F, and 93% hemoglobin S. Hydroxyurea therapy is found to be beneficial in this patient. An increase in which of the following is the most likely basis for its therapeutic efficacy? A Erythrocyte production B Overall globin chain synthesis C Oxygen affinity of hemoglobin D Production of hemoglobin A E Production of hemoglobin F

E Production of hemoglobin F Children and adults with sickle cell anemia may benefit from hydroxyurea therapy, which can increase the concentration of hemoglobin F in RBCs, which interferes with the polymerization of hemoglobin S. However, the therapeutic response to hydroxyurea often precedes the increase in hemoglobin F levels. Hydroxyurea also has an anti-inflammatory effect, increases the mean RBC volume, and can be oxidized by heme groups to produce nitric oxide that promotes vasodilation. Because hemoglobin F levels remain high through the first 5 to 6 months of life, patients with sickle cell anemia typically do not manifest the disease during infancy. Because both β-globin chains are affected, no hemoglobin A1 is produced, and A2 levels are never high. Globin synthesis overall is not going to increase, and globin synthesis must be balanced to produce normal hemoglobin. The hemolysis associated with sickling promotes erythropoiesis, but the concentration of hemoglobin S is not changed. Hydroxyurea does not significantly shift the oxygen dissociation curve or change the oxygen affinity of the various hemoglobins. (CH14)

A 55-year-old woman has felt an enlarging lump in her left breast for the past year. A hard, irregular 5-cm mass fixed to the underlying chest wall is palpable in her left breast. Left axillary nontender lymphadenopathy is noted. There is no hepatosplenomegaly. A chest CT scan reveals multiple bilateral pulmonary "cannonball" nodules. A left breast biopsy is performed, and on microscopic examination shows high-grade infiltrating ductal carcinoma. The appearance of the nodules in her lungs is most likely related to which of the following? A Internal mammary artery invasion by carcinoma cells B Lymphatic connections between the breast and the pleura C Overexpression of estrogen receptors within the carcinoma cell nuclei D Proximity of the breast carcinoma to the lungs E Pulmonary chemokines that bind carcinoma cell chemokine receptors

E Pulmonary chemokines that bind carcinoma cell chemokine receptors There is increasing evidence that localization of cancer metastases is influenced by the expression of chemo- kine receptors by cancer cells and elaboration of their ligands (chemokines) by certain tissues. In the case of breast cancer, the carcinoma cells express CXCR4 chemokines. Vascular, lymphatic, or basement membrane invasion is required for metastases, but these characteristics do not dictate accurately the location of metastases. (CH7)

In an experiment, release of epidermal growth factor into an area of denuded skin causes mitogenic stimulation of the skin epithelial cells. Which of the following proteins is most likely to be involved in transducing the mitogenic signal from the epidermal cell membrane to the nucleus? A Cyclic AMP B Cyclin D C Cyclin-dependent kinase D G proteins E RAS proteins

E RAS proteins RAS proteins transduce signals from growth factor re- ceptors, such as epidermal growth factor, that have intrinsic tyrosine kinase activity. G proteins perform a similar function for G protein-linked, seven-transmembrane receptors. Cyclic AMP is an effector in the G protein signaling pathway. Cyclins and cyclin-dependent kinases regulate the cell cycle in the nucleus. (CH1)

A 23-year-old woman, who works as a secretary for an accounting firm in Kiev, has noted a palpable nodule on the side of her neck for the past 3 months. On physical examination, there is a 2-cm, firm, nontender nodule involving the right lobe of the thyroid gland. Sequencing of DNA derived from the nodule shows rearrangement of the RET gene. No other family members are affected by this disorder. Which of the following findings would be considered most relevant in this woman's medical history? A Chronic dietary iodine deficiency B Chronic ethanol abuse C Congenital ataxia telangiectasia D Ingestion of arsenic compounds E Radiation exposure in childhood

E Radiation exposure in childhood Radiation is oncogenic, and the risk increases with higher dosages. Cancers of thyroid and bone as well as leukemias may develop years following environmental radiation exposure. Dosages of therapeutic radiation are carefully controlled, but risk for subsequent malignancy is still increased. The Chernobyl nuclear reactor disaster affecting persons in Belarus and Ukraine is a cautionary tale regarding environmental exposure to radiation. Trauma is not a risk factor for development of cancer, although traumatic episodes often are recalled and irrationally associated with subsequent health problems. Lack of iodine leads to goiter but not to thyroid neoplasia. Hepatocellular carcinomas can arise in cirrhosis caused by chronic alcoholism. Ataxia telangiectasia is an inherited syndrome that carries an in- creased risk of development of leukemias and lymphomas. Arsenic exposure, which is uncommon, leads to lung and skin cancers.

An epidemiologic study investigates the potential mor- phologic and molecular alterations that may contribute to the development of cancers in a population. Data analyzed from resected colonic lesions show changes that suggest the evolution of a sporadic colonic adenoma into an invasive carcinoma. Which of the following best describes the mechanism producing these changes leading to colonic malignancies? A Activation of proto-oncogenes by chromosomal translocation B Extensive regeneration of tissues increasing the mutation rate in regenerating cells C Inheritance of defects in TP53 genes that increase the susceptibility to develop cancer D Overexpression of growth factor receptor genes E Stepwise accumulation of multiple proto-oncogene and tumor suppressor gene mutations

E Stepwise accumulation of multiple proto-oncogene and tumor suppressor gene mutations Development of colonic adenocarcinoma typically takes years, during which time multiple mutations occur within the mucosa, including mutations involving such genes as APC (adenomatous polyposis coli), K-RAS, and p53. The accumulation of mutations, rather than their occurrence in a specific order, is most important in the development of a carcinoma. Activation of proto-oncogenes, extensive re- generation, faulty TP53 genes, and amplification of growth factor receptor genes all contribute to the development of malignancies, but they are not sufficient by themselves alone to produce a carcinoma from an adenoma of the colon. Inher- ited loss of wild-type TP53 contributes to multiple cancers, but not to sporadic adenomas. (CH7)

A 34-year-old woman reports becoming increasingly tired for the past 5 months. On physical examination, she is afebrile and has mild splenomegaly. Laboratory studies show a hemoglobin concentration of 10.7 g/dL and hematocrit of 32.3%. The peripheral blood smear shows spherocytes and rare nucleated RBCs. Direct and indirect Coombs test results are positive at 37° C, although not at 4° C. Which of the following underlying diseases is most likely to be diagnosed in this patient? A Escherichia coli septicemia B Hereditary spherocytosis C Infectious mononucleosis D Mycoplasma pneumoniae infection E Systemic lupus erythematosus

E Systemic lupus erythematosus This patient has a warm autoimmune hemolytic ane- mia secondary to systemic lupus erythematosus (SLE). A positive Coombs test result indicates the presence of anti- RBC antibodies in the serum and on the RBC surface. Most cases of warm autoimmune hemolytic anemia are idiopathic, but one fourth occur in individuals with an identifiable autoimmune disease, such as SLE; in other cases, drugs are the cause. The immunoglobulin coating the RBCs acts as an opsonin to promote splenic phagocytosis. Nucleated RBCs can be seen in active hemolysis because the marrow compensates by releasing immature RBCs. Septicemia is more likely to lead to a microangiopathic hemolytic anemia. The increased RBC destruction in hereditary spherocytosis is extravascular and not immune mediated. Infections such as mononucleosis and Mycoplasma are associated with cold autoimmune hemolytic anemia (with an elevated cold agglutinin titer). (CH14)

A 76-year-old man has experienced abdominal pain for the past year. On physical examination, there is an epigastric mass. An abdominal CT scan shows a 10-cm mass in the body of the pancreas. A fine-needle biopsy specimen of this mass shows a moderately differentiated adenocarcinoma. Mutational analysis of the carcinoma cells shows inactivation of cyclin-dependent kinase inhibitor with loss of growth-suppression. Regulatory pathways controlled by which of the following genes are most likely altered in this man's carcinoma? A BCL2 B β-Catenin C MYC D p53 E TGF-β

E TGF-β TGF-β inhibits cell proliferation by activation of growth-inhibiting genes, such as the CDKIs. All pancreatic cancers and 83% of colon cancers have at least one mutation- al event in a TGF-β pathway. The BCL2 family of genes acts as a regulator of apoptosis. The β-catenin pathway seen with the APC gene is involved with growth regulation; loss of the APC gene loci leads to failure in destruction of β-catenin, which translocates to the cell nucleus, where it functions as a transcription factor promoting growth. The MYC gene is a target of the activated RAS pathway. The p53 protein is in- volved in tumor suppression. (CH7)

In an experiment, cells from human malignant neoplasms explanted into tissue culture medium continue to replicate. This allows development of "immortal" tumor cell lines that are extremely useful for the study of tumor biology and responses to therapeutic modalities. Activation of which of the following molecular components is most likely to endow these tumor cells with limitless replicative ability in vivo and in vitro? A Hypoxia-induced factor 1 B BCL2 gene C Cyclin-dependent kinase gene methylation D DNA replication repair E Telomerase

E Telomerase Chromosomal telomere shortening in normal human cells limits their replicative potential and gives rise to replicative senescence. This occurs because most somatic cells lack the enzyme telomerase. Normal human stem cells do express telomerase. By contrast, 90% or more of human tumor cells show activation of telomerase, explaining continued tumor growth in the body and "immortalized" cell lines in culture. All other pathways listed cannot affect telomerase shortening, which is the rate-limiting step in indefinite replication of cells. (CH7)

A 42-year-old man is concerned about a darkly pigment- ed "mole" on the back of his hand. The lesion has enlarged and bled during the past month. On physical examination, there is a slightly raised, darkly pigmented, 1.2-cm lesion on the dorsum of the right hand. The lesion is completely excised. The microscopic appearance is shown in the figure. Which of the following factors presents the greatest risk for the development of this neoplasm? A Allergy to latex B Asbestos exposure C Chemotherapy D Smoking tobacco E Ultraviolet radiation

E Ultraviolet radiation Worldwide, increasing numbers of skin cancers occur because of sun exposure. The ultraviolet light damages the skin and damages cellular DNA, leading to mutations that can escape cellular repair mechanisms. Allergic reactions do not promote cancer. Asbestos exposure increases lung carci- noma risk in smokers and can lead to rare mesotheliomas of pleura. Chemotherapeutic agents have carcinogenic poten- tial, particularly alkylating agents such as cyclophosphamide, but leukemias and lymphomas are the usual result. Smoking tobacco is related to many cancers, but skin cancers are not typically associated with this risk factor. (CH7)

A 77-year-old woman notices that small, pinpoint-to- blotchy areas of superficial hemorrhage have appeared on her gums and on the skin of her arms and legs over the past 3 weeks. On physical examination, she is afebrile and has no organomegaly. Laboratory studies show a normal prothrombin time and partial thromboplastin time. CBC shows hemoglobin of 12.7 g/dL, hematocrit of 37.2%, MCV of 80 μm3, platelet count of 276,000/mm3, and WBC count of 5600/mm3. Platelet function studies and fibrinogen level are normal, and no fibrin split products are detectable. Which of the following conditions best explains these findings? A Chronic renal failure B Macronodular cirrhosis C Meningococcemia D Metastatic carcinoma E Vitamin C deficiency

E Vitamin C deficiency Platelet number and function in this case are normal, and there is no detectable abnormality in the extrinsic or intrinsic pathways of coagulation as measured by the pro- thrombin time or partial thromboplastin time. Petechiae and ecchymoses can result from increased vascular fragility, a consequence of nutritional deficiency (e.g., vitamin C), infec- tion (e.g., meningococcemia), and vasculitic diseases. Chron- ic renal failure may depress platelet function. Chronic liver disease would affect the prothrombin time. Meningococce- mia is an acute illness. Metastatic disease does not directly affect hemostasis, although extensive marrow metastases could diminish platelet production. (CH14)

A 22-year-old man has a raised, pigmented lesion on his forearm that has increased in size and become more irregular in color over the past 4 months. Physical examination shows a 0.5 × 1.2 cm black-to-brown asymmetric lesion with irregular borders. An excisional biopsy specimen shows clusters of pleomorphic pigmented cells that extend into the reticular dermis. Family history indicates that the patient's maternal uncle died from a similar tumor. His grandfather required enucleation of the left eye because of a "dark brown" retinal mass. Which of the following genes is most likely to have undergone mutation to produce these findings in this family? A BCL2 (anti-apoptosis gene) B c-MYC (transcription factor gene) C IL2 (growth factor gene) D Lyn (tyrosine kinase gene) E p16 (cell cycle inhibition) F p53 (DNA damage response gene)

E p16 (cell cycle inhibition) A family history of malignant melanoma is pres- ent. Familial tumors often are associated with inheritance of a defective copy of one of several tumor suppressor genes. In the case of melanomas, the implicated gene is called p16, or INK4a. The product of the p16 gene is an inhibitor of cy- clin-dependent kinases. Germline mutations in CDKN2A may also underlie familial melanomas. With loss of control over cyclin-dependent kinases, the cell cycle cannot be regu- lated, favoring neoplastic transformation. BCL2 is present in some lymphoid neoplasms. The c-MYC gene is mutated in various carcinomas, but is not known to be specifically asso- ciated with melanomas. The IL2 mutation is associated with some T cell neoplasms. The Lyn mutation is seen in some im- munodeficiency states. p53 mutations occur in many cancers, but not specifically in familial melanomas. (CH7)

A 63-year-old man has a cough with hemoptysis for 10 days. He has a 65 pack-year history of smoking. A chest CT scan shows a 5-cm right hilar mass. Bronchoscopy is performed, and lung biopsy specimens show small cell anaplastic lung carcinoma. His family history shows three first-degree maternal relatives who developed leukemia, sarcoma, and carcinoma before age 40 years. Which of the following gene products is most likely to have been altered by mutation to produce these findings? A APC (tumor suppressor) B BCL2 (anti-apoptosis) C K-RAS (GTP binding) D NF1 (GTPase activation) E p53 (DNA damage response)

E p53 (DNA damage response) p53 is the most common target for genetic altera- tions in human neoplasms. Most are sporadic mutations, although some are inherited. The inheritance of one faulty p53 suppressor gene predisposes to a "second hit" that eliminates the remaining p53 gene. Homozygous loss of the p53 genes dysregulates the repair of damaged DNA, predisposing individuals to multiple tumors, as in this case. The APC gene is mutated in sporadic colon cancers and in familial polyposis coli. The BCL2 gene is mutated in some non-Hodgkin lymphomas. The HER2 gene is one of the EGF receptor family members amplified in some breast cancers. The EGF mutation is most often seen in squamous cell carcinomas of the lung. K-RAS mutations are present in many cancers, but not typically in lymphoid malignancies. The NF1 gene mutation is seen in neurofibromatosis type 1. (CH7)

The mother of a 5-year-old boy notices that his abdomen has enlarged in the past 6 months. On physical examination there is an ill-defined abdominal mass. An abdominal CT scan shows a 9-cm mass in the region of the right adrenal gland. The mass is removed and microscopically shows primitive hyperchromatic cells. Cytogenetic analysis of tumor cells shows many double minutes and homogeneously staining regions. Which of the following genes is most likely to have undergone alterations to produce these findings? A BCL1 (cyclin gene) B BCL2 (anti-apoptosis gene) C IL2 (growth factor gene) D K-RAS (GTP-binding protein gene) E Lyn (tyrosine kinase gene) F N-MYC (transcription factor gene)

F N-MYC (transcription factor gene) Double minutes and homogeneously staining regions seen on a karyotype represent gene amplifications. Amplification of the N-MYC gene occurs in 30% to 40% of neuroblastomas, and this change is associated with a poor prognosis. The BCL1 and BCL2 genes are mutated in some non-Hodgkin lymphomas. The IL2 mutation may be present in some T cell neoplasms. K-RAS mutations are present in many cancers, but not typically childhood neoplasms. The Lyn mutation is seen in some immunodeficiency states. (CH7)

A 30-year-old, previously healthy man from Lagos, Nigeria, passes dark brown urine 2 days after starting the prophylactic antimalarial drug primaquine. On physical examination, he appears pale and is afebrile. There is no organomegaly. Laboratory studies show that his serum haptoglobin level is decreased. Which of the following is the most likely explanation of these findings? A Antibody-mediated hemolysis B Impaired DNA synthesis C Impaired globin chain synthesis D Increased susceptibility to complement-induced lysis E Mechanical fragmentation of RBCs as a result of vascular narrowing F Oxidative injury to hemoglobin G Reduced deformability of RBC membrane

F Oxidative injury to hemoglobin Glucose-6-phosphate dehydrogenase (G6PD) deficiency predisposes the hemoglobin in RBCs to oxidative injury from drugs such as primaquine, and can induce hemolysis. Oxidant injury to hemoglobin produces inclusion of denatured hemoglobin within RBCs. The inclusions damage the cell membrane directly, giving rise to intravascular hemolysis. These damaged RBCs have reduced membrane deformability, and they are removed from the circulation by the spleen. The remaining mechanisms listed are not directly drug dependent. Hemolytic anemias with antibody coating RBCs can occur with autoimmune diseases, prior transfusion, and erythroblastosis fetalis. Impaired RBC nuclear maturation occurs as a result of vitamin B12 or folate deficiency. Impaired globin synthesis occurs in thalassemias. Complement lysis is enhanced in paroxysmal nocturnal hemoglobinuria, which results from mutations in the PIGA gene. Mechanical fragmentation of RBCs is typical of microangiopathic hemolytic anemias, such as disseminated intravascular coagulation. Reduced RBC membrane deformability is seen in patients with abnormalities in cytoskeletal proteins, such as spectrin; the latter causes hereditary spherocytosis. (CH14)

A 30-year-old man has a pheochromocytoma of the left adrenal gland; a sibling had a cerebellar hemangioblastoma. He undergoes adrenalectomy, and on microscopic examination there is extensive vascularity of the neoplasm. Mutational analysis of the neoplastic cells shows that both allelic copies of a gene have been lost, so that a protein that binds to hypoxia-inducible factor 1-alpha is no longer ubiquitinated, but instead translocates to the nucleus and activates transcription of VEGF. Which of the following genes is most likely mutated in this man? A APC B BCL2 C EGF D HER2 E HST1 F MYC G VHL

G VHL Angiogenesis is a key feature of neoplasms because the growing tumor needs a blood supply, and up-regulation of factors such as VEGF and FGF help to keep the cancer growing. VEGF may be up-regulated by activation of hypoxia-inducible factor 1-alpha (HIF-1-alpha). The von Hippel-Lindau (VHL) gene acts as a tumor suppressor, and it normally produces a protein that binds to hypoxia-inducible factor 1-alpha so that it is cleared. VHL mutation leads to loss of this binding protein and activation of angiogenesis factors. Individuals with VHL syndrome have various neoplasms, including pheochromocytomas, renal cell carcinomas, and hemangioblastomas. The other listed genes have products that do not directly act on angiogenesis pathways. (CH7)

A 28-year-old woman has had a constant feeling of lethargy since childhood. On physical examination, she is afebrile and has a pulse of 80/min, respirations of 15/min, and blood pressure of 110/70 mm Hg. The spleen tip is palpable, but there is no abdominal pain or tenderness. Laboratory studies show hemoglobin of 11.7 g/dL, platelet count of 159,000/ mm3, and WBC count of 5390/mm3. The peripheral blood smear shows small round erythrocytes that lack a zone of central pallor. An inherited abnormality in which of the following RBC components best accounts for these findings? A α-Globin chain B β-Globin chain C Carbonic anhydrase D Glucose-6-phosphate dehydrogenase E Heme with porphyrin ring F Spectrin cytoskeletal protein

F Spectrin cytoskeletal protein Hereditary spherocytosis is a condition in which a mutation affects one of several membrane cytoskeletal proteins. Spectrin and related proteins are cytoskeletal proteins that are important in maintaining the RBC shape. These proteins include ankyrin (most common) and band 4.2, which binds spectrin to the transmembrane ion transporter; band 3; and protein 4.1, which binds the "tail" of spectrin to another transmembrane protein, glycophorin A. The abnormal RBCs with such mutant proteins are less deformable, lack central pallor on a peripheral blood smear, and they are sequestered and destroyed in the spleen. Thalassemias with abnormal α-globin or β-globin chains are associated with hypochromic microcytic anemias. Iron deficiency affects the heme portion of hemoglobin, leading to hypochromia and to microcytosis. Carbonic anhydrase in RBCs helps to maintain buffering capacity. Glucose-6-phosphate dehydrogenase (G6PD) deficiency is an X-linked condition that most commonly affects black males. Porphyrias may affect the production of porphyrin rings and may lead to hemolytic anemia along with abdominal pain, neurologic problems, or skin findings. (CH14)

A Pap smear obtained from a 29-year-old woman during a routine health maintenance examination is abnormal. She is currently asymptomatic. She has a history of multiple sexual partners. Cervical biopsy specimens are obtained and the microscopic appearance is shown in the figure. Which of the following is the most likely diagnosis? A Adenocarcinoma B Carcinoma in situ C Dysplasia D Hamartoma E Metaplastic transformation F Squamous cell carcinoma

F Squamous cell carcinoma In the figure the disorderly, atypical epithelial cells involve the entire thickness of the epithelium. They extend through the underlying basement membrane and into the underlying stroma as rounded nests at the right, a process known as invasion. The ectocervix and the squamous meta- plasia of endocervix give rise to dysplasia from which squa- mous cell carcinoma can arise. Carcinoma in situ is confined to the epithelium; if the basement membrane is breached, the lesion is no longer in situ, but rather invasive. An ade- nocarcinoma is a malignant neoplasm arising from glandu- lar epithelium, such as the endocervix or endometrium, not the ectocervix. A dysplastic process could precede develop- ment of carcinoma in situ and squamous carcinoma; dys- plasia involves only part of the thickness of the epithelium. A hamartoma contains a mixture of cell types common to a tissue site. Metaplasia can occur in response to persistent infection with human papillomavirus (HPV) and other in- flammatory conditions. Metaplasia can be the precursor to dysplasia. (CH7)

An experiment is conducted in which cells in tissue culture are subjected to high levels of ultraviolet radiant energy. Electron microscopy shows cellular damage in the form of increased cytosolic aggregates of denatured proteins. In situ hybridization reveals that protein components in these aggregates also are found in proteasomes. Which of the following substances most likely binds to the denatured proteins, targeting them for catabolism by cytosolic proteasomes? A Adenosine monophosphate B Calcium C Caspase D Granzyme B E Hydrogen peroxide F Ubiquitin

F Ubiquitin Heat-shock proteins provide for a variety of cellular "housekeeping" activities, including recycling and restora- tion of damaged proteins and removal of denatured pro- teins. Ubiquitin targets denatured proteins and facilitates their binding to proteasomes, which then break down the proteins to peptides. ADP increases when ATP is depleted, helping to drive anaerobic glycolysis. Cytosolic calcium lev- els may increase with cell injury that depletes ATP; the calci- um activates phospholipases, endonucleases, and proteases, which damage the cell membranes, structural proteins, and mitochondria. Caspases are enzymes that facilitate apopto- sis. Granzyme B is released from cytotoxic T lymphocytes and triggers apoptosis. Hydrogen peroxide is one of the acti- vated oxygen species generated under conditions of cellular ischemia, producing nonspecific damage to cellular struc- tures, particularly membranes. (CH1)

A 51-year-old man who works in a factory that produces plastic pipe has experienced weight loss, nausea, and vomiting over the past 4 months. On physical examination, he has tenderness to palpation in the right upper quadrant of the abdomen, and the liver span is increased. Laboratory findings include serum alkaline phosphatase, 405 U/L; AST, 45 U/L; ALT, 30 U/L; and total bilirubin, 0.9 mg/dL. An abdominal CT scan shows a 12-cm mass in the right lobe of the liver. A liver biopsy is performed, and microscopic examination shows a malignant neoplasm of endothelial cells. The patient has most likely been exposed to which of the following agents? A Arsenic B Asbestos C Benzene D Beryllium E Nickel F Vinyl chloride G Naphthalene

F Vinyl chloride Vinyl chloride is a rare cause of liver cancer. This causal relationship was easy to show, however, because hepatic angiosarcoma is a rare neoplasm. Arsenic is a risk factor for skin cancer. Asbestos exposure is linked to pleural malignant mesothelioma and to bronchogenic carcinomas in smokers. Benzene exposure is linked to leukemias. Beryllium exposure can produce interstitial lung disease and lung cancer. Nickel exposure increases the risk of respiratory tract cancers. Exposure to naphthalene compounds is a risk factor for cancers of the urinary tract. (CH7)

A 3-year-old child has exhibited difficulty with vision in her right eye. On physical examination, there is leukocoria of the right eye, consistent with a mass in the posterior chamber. MR imaging shows a mass that nearly fills the globe. The child undergoes enucleation of the right eye. Molecular analysis of the neoplastic cells indicates absence of both copies of a gene that contributes to control of the cell cycle. Which of the following genes has most likely undergone mutation in this neoplasm? A BCR-ABL B BCL2 C hMSH2 D K-RAS E NF1 F p53 G RB

G RB The RB gene is a classic example of the two-hit mechanism for loss of tumor suppression. About 60% of these tumors are sporadic, whereas the rest are familial from inheritance of a mutated copy of the RB gene. Loss of the second copy in retinoblasts leads to the occurrence of reti- noblastoma in childhood. The RB gene controls the G1 to S transition of the cell cycle; with loss of both copies, this im- portant checkpoint in the cell cycle is lost. The BCR-ABL fusion gene in chronic myelogenous leukemia is an example of overexpression of a gene product producing neoplasia. The BCL2 gene is an inhibitor of apoptosis. The hMSH2 gene is present in most cases of hereditary nonpolyposis colon cancer and functions in DNA repair. Many cancers have the K-RAS gene, which acts as an oncogene. The NF1 gene product acts as a tumor suppressor; this is a component of neurofibromatosis (which usually does not involve the eye), and the neoplasms typically appear at a later age. Many cancers have the p53 tumor suppressor gene mutation, but this is not typical of childhood ocular neoplasms. (CH7)

A 32-year-old woman has experienced dull pelvic pain for the past 2 months. Physical examination shows a right adnexal mass. An abdominal ultrasound scan shows a 7.5-cm cystic ovarian mass. The mass is surgically excised. The surface of the mass is smooth, and it is not adherent to surrounding pelvic structures. On gross examination, the cystic mass is filled with hair. Microscopically, squamous epithelium, tall columnar glandular epithelium, cartilage, and fibrous connective tissue are present and resemble normal tissue counter- parts. Which of the following is the most likely diagnosis? A Adenocarcinoma B Fibroadenoma C Glioma D Hamartoma E Mesothelioma F Rhabdomyosarcoma G Teratoma

G Teratoma A teratoma is a neoplasm derived from totipotential germ cells that differentiate into tissues that represent all three germ layers: ectoderm, endoderm, and mesoderm. When the elements all are well differentiated, the neoplasm is "mature" (benign). Adenocarcinomas have malignant- appearing glandular elements. Fibroadenomas have a be- nign glandular and stromal component; they are common in the breast. Gliomas are found in the central nervous system. Hamartomas contain a mixture of cell types common to a tissue site; the lung is one site for this uncommon lesion. A mesothelioma arises from the lining of thoracic and abdomi- nal body cavities. A rhabdomyosarcoma comprises cells that poorly resemble striated muscle; most arise in soft tissues. (CH7)


Ensembles d'études connexes

Human Nutrition Chapter 4 quiz review

View Set

Life Insurance Test (completing the application, underwriting, and delivering the policy)

View Set

Aceable Agent - Law of Contracts

View Set

Mc Graw Hill: Vocabulary Review Africa

View Set

Muscle and Nervous Tissue Microanatomy

View Set

Unit Test: 4.13 Unit Test: Chemical Thermodynamics

View Set

Government Inquizitive Ch 4: Civil Liberties and Civil Rights

View Set

LQ6--Sagittal Brain(fig 13.9) CHAP-13 Quiz 10

View Set